Jan 31

You might also like

Download as pdf or txt
Download as pdf or txt
You are on page 1of 66

PARADISE WEEKLY MODEL TEST CEE MD/MS BASED MAGH 16

PARADISE WEEKLY MODEL TEST


CEE MD/MS BASED MODEL TEST
(Tuesday, Magh 16, 2080)
SCAN THE QR CODE AND JOIN THE COMMUNITY OF PG ASPIRANTS IN VIBER
- Discuss with fellow aspirants.
- Get notices about MD/MS Entrances
- Get notices about mock tests, results and offers
- Ask confusing questions to the experts
- Scroll the groups, and learn.

Questions Answers Explanations (QAE)

Medicine

1. A 19-year-old woman arrives at the ED with a 15-hour history of abdominal pain, nausea, and vomiting. She was
awoken early in morning by severe abdominal pain. She does admit to drinking heavily the prior evening that is not
unusual during the weekends. She does not use NSAIDs regularly. Her BP is 100/60 mm Hg and her PR 130
beats/min, RR 14 breaths/min, her Temp 39°C. Acute abdominal series upon admission displayed substantial
amount of free air under the right hemidiaphragm. Which of the following is the most likely diagnosis?
a. Perforated peptic ulcer
b. Alcohol-relatedgastritis
c. Appendicitis
d. Gastroenteritis

Ans: a
Ref: Talley NJ. American Gastroenterological Association medical position statement:evaluation of dyspepsia.
Gastroenterology. 2005;129(5):1753-1755.
The acute abdomen and free air under the diaphragm indicates a perforated viscus. This patient has perforated ulcer with
hemody- namic instability. Additional workup includes a chemistry panel, CBC, and urgent laparotomy.

2. A 34-year-old man is brought into the ED after a motor vehicle accident. He complains of dyspnea and initially
had an oxygen saturation of 88%. O/E, he has decreased breath sounds of the right chest and now has an oxygen
saturation of 70% on room air. Which of the following is the most appropriate next step?
a. Chest radiograph
b. CT of the chest
c. Tube thoracostomy
d. Heparin anticoagulation

Ans: c
Ref: Harrison’s Principles of Internal Medicine; 20th Edition
The constellation of clinical signs points toward a pneumothorax. The pres- ence of significant hypoxia requires immediate
placement of a chest tube prior to chest radiograph confirmation as further delay may progress to cardiovascular collapse.

3. A 30yrs female has history of migraine headache for the past 2yrs and has been under over-the-counter
analgesics. Over the past few days, she feels lethargic and complains of mild headache and rhinorrhea. The most
common cause of secondary headache will likely be;
a. Systemic infection
b. Tension type

Paradise Institute & Self Study Centre Page 1


PARADISE WEEKLY MODEL TEST CEE MD/MS BASED MAGH 16

c. Migraine
d. Cluster

Ans: a
Ref: Harrison’s Principles of Internal Medicine; 20th Edition, Page No: 85

4. A 35-year-old man has been involved in aRTA, and is found to be hypotensive. Which of the following locations of
bleeding can cause significant complications but does not explain the hypotension?
a. Chest and abdomen
b. Pelvic girdle and soft-tissue compartments
c. External bleeding
d. Intracranial bleeding

Ans: d
Ref: Harrison’s Principles of Internal Medicine; 20th Edition
It is important to systematically check for bleeding sources in the chest, abdomen, pelvic girdle, soft-tissue compartments
(long-bone fractures), and external bleeding. Intracranial bleeding, although a significant injury, is usually not the cause of
hypotension. The exception to this is the patient who is mori- bund secondary to a head injury.

5. A 24-year-old patient has 4 days history of fever and cold. Crept are heard in bilateral lower chest regions.
Neutrophil is the dominant cell type in alveolar space. Which of the following is the pathological phase of his
pneumonia?
a. Edema
b. Red hepatization
c. Gray hepatization
d. Resolution

Ans: c
Ref: Harrison 20th Edition, Page No: 909
Classic pneumonia evolves through a series of pathologic changes.
• The initial phase is one of edema, with the presence of a proteinaceous exudate– and often of bacteria–in the
alveoli.
• This phase is rarely evident in clinical or autopsy specimens because of the rapid transition to the red hepatization
phase. The presence of erythrocytes in the cellular intraalveolar exudate gives this second stage its name, but
neutrophil influx is more important with regard to host defense. Bacteria are occasionally seen in pathologic
specimens collected during this phase.
• In the third phase, gray hepatization, no new erythrocytes are extravasating, and those already present have been
lysed and degraded. The neutrophil is the predominant cell, fibrin deposition is abundant, and bacteria have
disappeared. This phase corresponds with successful containment of the infection and improvement in gas
exchange.
• In the final phase, resolution, the macrophage reappears as the dominant cell type in the alveolar space, and the
debris of neutrophils, bacteria, and fibrin has been cleared, as has the inflammatory response.

6. Which of the following clinical picture dominate in Eosinophilic granulomatosis with polyangiitis
Paradise Institute & Self Study Centre Page 2
PARADISE WEEKLY MODEL TEST CEE MD/MS BASED MAGH 16

a. Asthma and pulmonary infiltrates


b. Mononeuritis multiplex
c. Glomerulonephritis
d. Oral and nasal ulcer

Ans: a
Ref: Harrison’s Principles of Internal Medicine; 20th Edition, Page No: 2582
 Patients with eosinophilic granulomatosis with polyangiitis (Churg-Strauss) often exhibit nonspecific
manifestations such as fever, malaise, anorexia, and weight loss, which are characteristic of a multisystem disease.
 The pulmonary findings in eosinophilic granulomatosis with polyangiitis (Churg-Strauss) clearly dominate the
clinical picture with severe asthmatic attacks and the presence of pulmonary infiltrates.
 Mononeuritis multiplex is the second most common manifestation and occurs in up to 72% of patients. Allergic
rhinitis and sinusitis develop in up to 61% of patients and are often observed early in the course of disease.
 Clinically recognizable heart disease occurs in ~14% of patients and is an important cause of mortality. Skin
lesions occur in ~51% of patients and include purpura in addition to cutaneous and subcutaneous nodules.
 The renal disease in eosinophilic granulomatosis with polyangiitis (Churg-Strauss) is less common and generally
less severe than that of granulomatosis with polyangiitis and microscopic polyangiitis.

7. A 34-year-old man admits taking “the whole bottle” of acetaminophen over the course of 36 hours because of a
severe headache. Which of the following is the best guide to determine whether to initiate NAC therapy?
a. Initiate NAC due to potentially toxic exposure
b. Serum APAP level and liver enzymes
c. Plotting the serum APAP level on the nomogram
d. If over 24 hours have elapsed, NAC therapy is not efficacious

Ans: a
Ref: Essentials of Emergency Medicine. Sudbury, MA: Jones and Bartlett; 2006:792-794.
At 24 hours postingestion, NAC therapy may still be useful. Due to the historically toxic exposure, NAC should be started
while a serum APAP level and liver enzymes are checked. If the APAP level is undetectable and the liver enzymes are
normal, subsequent doses of NAC can be discontinued. The Rumack-Matthew nomogram is not applicable for ingestions
more than 24 hours prior to evaluation.

8. Drug of choice in treatment of attack of acute intermittent porphyria is


a. IV hemin
b. Chloral hydrate
c. IV glucose
d. Orthotopic liver transplantation

Ans: a
Ref: Harrison’s Principles of Internal Medicine; 20th Edition, Page No: 2993
 During acute attacks, narcotic analgesics may be required for abdominal pain, and phenothiazines are useful for
nausea, vomiting, anxiety, and restlessness.
 Chloral hydrate can be given for insomnia, and benzodiazepines are probably safe in low doses if a minor
tranquilizer is required.
 Carbohydrate loading, usually with intravenous glucose (at least 300 g daily), may be effective in milder acute
attacks of porphyria (without paresis, hyponatremia, etc.) if hemin is not available. Intravenous hemin is more
effective and should be used as first-line therapy for all acute attacks.
 Orthotopic liver transplantation (OLT) has been successful and is curative in patients with severe, disabling,
intractable attacks that are refractory to hemin therapy.

9. Excruciating paroxysms of pain in the lips, gums, cheek with pain lasting for more than a few seconds or a
minute or two but may be so intense that the patient winces and sometimes patient complains of pain generated
while washing face or brushing the teeth is seen with:
a. Temporal arteritis
Paradise Institute & Self Study Centre Page 3
PARADISE WEEKLY MODEL TEST CEE MD/MS BASED MAGH 16

b. Trigeminal neuralgia
c. Migraine
d. Cluster tic

Ans: b
Ref: Harrison’s Principles of Internal Medicine; 20th Edition, Page No: 3167
 Trigeminal neuralgia is characterized by excruciating paroxysms of pain in the lips, gums, cheek, or chin and, very
rarely, in the distribution of the ophthalmic division of the fifth nerve.
 The pain seldom lasts more than a few seconds or a minute or two but may be so intense that the patient winces,
hence the term tic. The paroxysms, experienced as single jabs or clusters, tend to recur frequently, both day and
night, for several weeks at a time.
 They may occur spontaneously or with movements of affected areas evoked by speaking, chewing, or smiling.
 Another characteristic feature is the presence of trigger zones, typically on the face, lips, or tongue, that provoke
attacks; patients may report that tactile stimuli—e.g., washing the face, brushing the teeth, or exposure to a draft of
air—generate excruciating pain.
 An essential feature of trigeminal neuralgia is that objective signs of sensory loss cannot be demonstrated on
examination.

10. A 32-year-old trekker wandered into the abandoned trekking track and lost his bearings. When he was finally
found, he was brought to the ED with severe frostbite of the fingers of both hands. Initially, they appeared very
blue, but are beginning to look black. He has not regained sensation 24-hours after the exposure. Which of the
following describes the most appropriate time to wait before deciding on amputation of the affected fingers?
a. 24 hours after the episode
b. 48 to 72 hours after the episode
c. 7 to 10 days after the episode
d. 2 to 3 months after demarcation

Ans: d
Ref: Harrison’s Principles of Internal Medicine; 20th Edition, Page No: 2412
Generally, 3 weeks is the minimum time required to assess the viability of tissue after frostbite to see whether amputation
is required. Tissue thought to be necrotic sometimes turns out to be viable. The line of demarcation between viable and
nonviable tissue becomes clear in 1 to 2 months after the initial cold injury, but surgery may be delayed until 2 to 3
months.

11. The type of anemia most commonly seen in bacterial overgrowth syndrome is
a. Microcytic hypochromic
b. Macrocytic
c. Normocytic normochromic
d. None of the above

Ans: b
Reference: Uptodate,Small intestinal bacterial overgrowth: Clinical manifestation and diagnosis, Author:Mark
Pimentel, MD, FRCP(C) et al
 Laboratory abnormalities are usually associated with severe bacterial overgrowth or when SIBO occurs in
association with an underlying anatomic abnormality.
 Laboratory findings including macrocytic anemia, B12 deficiency, and the presence of fecal fat. Patients may also
have low levels of thiamine and niacin and elevated serum folate and vitamin K levels.
 In rare cases, microcytic anemia results from bleeding due to ulcers in stagnant intestinal loops or associated with
ileitis or colitis.
 Patients with protein losing enteropathy due to SIBO have hypoalbuminemia.

12. Aspirin is recommended in patient with Diabetes mellitus if


a. Age>50 years even without any cardiovascular risk
b. Age>50 years even with any one cardiovascular risk factors
Paradise Institute & Self Study Centre Page 4
PARADISE WEEKLY MODEL TEST CEE MD/MS BASED MAGH 16

c. Age>50 years even with at least any two cardiovascular risk factors
d. Contraindicated in patient with DM and age >50 years

Ans: b
Ref: Harrison’s Principles of Internal Medicine; 20th Edition, Page No: 2881
 Antiplatelet therapy reduces cardiovascular events in individuals with DM who have CHD and is recommended.
 The ADA recommends considering the use of aspirin for primary prevention of coronary events in individuals
with diabetes with an increased cardiovascular risk (>50 years with at least one risk factor such as hypertension,
dyslipidemia, smoking, family history, or albuminuria).
 ASA is not recommended for primary prevention in those with a low cardiovascular risk (<50 years with no risk
factors).
 The aspirin dose is the same as in nondiabetic individuals.

13. In heart failure, aldosterone secretion is elevated due to?


a. Prolongation of biological half life
b. Increased secretion
c. Reduced hepatic catabolism
d. All of the above

Ans: d
Ref: Harrison’s Principles of Internal Medicine; 19th Edition, Page No: 250
In patients with heart failure, not only is aldosterone secretion elevated but the biologic half-life of the hormone is
prolonged secondary to the depression of hepatic blood flow, which reduces its hepatic catabolism and increases further the
plasma level of the hormone.

14. HRCT imaging of a person working in glass manufacturing shows characteristic "crazy paving". This pattern is
characteristic of
a. Asbestosis
b. Coal worker's pneumoconiosis
c. Chronic Beryllium disease
d. Silicosis

Ans: d
Ref: Harrison’s Principles of Internal Medicine; 20th Edition, Page No: 1978-79
 Despite being one of the oldest known occupational pulmonary hazards, free silica (SiO2), or crystalline quartz, is
still a major cause of disease. The major occupational exposures include mining; stonecutting; sand blasting; glass
and cement manufacturing; foundry work; packing of silica flour; and quarrying, particularly of granite.
 The clinical and pathologic features of acute silicosis are similar to those of pulmonary alveolar proteinosis. The
chest radiograph may show profuse miliary infiltration or consolidation, and there is a characteristic HRCT pattern
known as “crazy paving”. The disease may be quite severe and progressive despite the discontinuation of exposure.

15. A34-year-old woman is brought into the ED for“ the worst headache of her life.” She has some lethargy,
photophobia, and nuchal rigidity. A lumbar puncture is performed after examining her eye grounds. Which of the
following findings in cerebrospinal fluid is most concerning for subarachnoid hemorrhage?
a. Red blood cells
b. White blood cells
c. Elevated opening pressure
d. Xanthochromia

Ans: d
Ref: Rosen’s Emergency Medicine: Concepts and Clinical Practice. 7th ed.
Xanthochromia in cerebrospinal fluid is most concerning for subarachnoid hemorrhage. Because it results from
hemoglobin metabolism, xanthochromia may take up to 12 hours to develop.

Paradise Institute & Self Study Centre Page 5


PARADISE WEEKLY MODEL TEST CEE MD/MS BASED MAGH 16

16. A Renal transplant recipient was under maintenance immunosuppressive drug. He developed hypertension,
dyslipidemia, nephrotoxicity and diabetes mellitus. The offending drug is probably
a. Sirolimus
b. Cyclosporine
c. Tacrolimus
d. Glucocorticoid

Ans: c
Ref: Harrison’s Principles of Internal Medicine; 20th Edition, Page No: 2128

17. A 33-year-old man is found comatose at a construction site in QATAR on a hot summer day. His core
temperature is 41.7°C (107°F). The ED physician orders evaporative cooling measures and ice packs. The patient
begins with intense shivering. Which of the following is the best next step?
a. Continued observation.
b. Short-acting benzodiazepine.
c. Increase the number of ice bags.
d. Stop the cooling.

Ans: b
Ref: Becker JA, Stewart LK. Heat-related illness. Am Fam Physician. 2011;83:1325-1330.
This patient most likely has exertional heatstroke where core temperature elevations may occur rapidly; therefore,
measurement directed at reducing his core temperature are appropriate and must be continued. Benzodiazepines are first-
line therapy for shivering or seizures in heat stroke.

18. A 22-year-old player comes to the ED complaining of 10 hours of severe right testicular pain. He denies a history
of trauma. On examination, his right testis is diffusely tender and indurated, and the pain does not change with
patient position. He has a cremasteric reflex on the right side. Which of the following is the best next step?
a. Continued observation
b. Oral antibiotics
c. Bed rest, ice to scrotum, and elevation of the scrotum
d. Surgical exploration of the scrotum

Ans: d
Ref: Harrison’s Principles of Internal Medicine; 20th Edition,
The clinical history is consistent with testicular torsion. The presence of a cremasteric reflex does not rule out the disease.
Emergency scrotal exploration is the procedure of choice when the history, physical examination, and imag- ing tests do
not rule out testicular torsion.

Surgery

Paradise Institute & Self Study Centre Page 6


PARADISE WEEKLY MODEL TEST CEE MD/MS BASED MAGH 16

19. A 32-year-old man was involved in a knife fight and had stab injuries to his abdomen, although it is unclear how
deep these injuries are. He is brought into the emergency room with a heart rate of 110 /min and BP 84/50 mm Hg.
Based on the clinical assessment, which of the following is the amount of acute blood loss he has experienced?
a. 250 mL
b. 500 mL
c. 1000 mL
d. 1500 Ml

Ans: d
Ref.:Mullins RJ. Management of shock. In: Mattox KL, Feliciano DV, Moore EE, eds. Trauma. New York, NY:
McGraw-Hill; 1999:195-234.
Blood pressure at rest typically does not decrease until class III hemorrhagic shock, when 1500 to 2000 mL of blood is lost
(30%-40% of blood volume). Class I hemorrhagic shock is well compensated associated with 750 mL EBL or less, with no
effect on blood pressure and minimal effect on heart rate. Class II shock, associated with 750 to 1500 mL EBL, is
associated with tachycardia but normal blood pressure at rest, and low urine output.

20. Which of the following is an advantage of the FAST examination in a patient with hemorrhagic shock?
a. Can identify retroperitoneal hematomas.
b. Can be performed quickly at bedside.
c. Can identify the specific site of injury.
d. Can quantify the exact amount of blood loss.

Ans: b
Ref: Mullins RJ. Management of shock. In: Mattox KL, Feliciano DV, Moore EE, eds. Trauma. New York, NY:
McGraw-Hill; 1999:195-234.
DPL and FAST cannot rule out retroperitoneal injury or identify the specific site of injury, but they can be performed
quickly at bedside on unstable trauma patients. To find the specific site of injury and rule out retroperitoneal injury, a CT
scan can be done; however, the trauma patient must be hemodynamically stable to be transported to the CT scan suite.

21. A 35-year-old male with history of RTA can propel power wheelchair, possibly push manual wheelchair,
transfer with assistance, dress upper body (lower body with assistance), self-groom with aids, bladder/bowel care
with assistance, self-feed with splints, able to drive. What could be the possible level of spinal cord injury?
a. C2
b. C4
c. C6
d. L1-L5

Ans: c
Ref: Bailey and Love’s Short Practice of Surgery 27th Edition, Page No: 353
Expected functional outcome versus level of cervical spinal cord injury
 C3–C4 Power wheelchair with mouth or chin control. Verbalise care, communicate through adaptive equipment.
May be ventilator dependent
Paradise Institute & Self Study Centre Page 7
PARADISE WEEKLY MODEL TEST CEE MD/MS BASED MAGH 16

 C5 Power wheelchair, dress upper body, self-feed with aids, wash face with assistance
 C6 Propel power wheelchair, possibly push manual wheelchair, transfer with assistance, dress upper body (lower
body with assistance), self-groom with aids, bladder/bowel care with assistance, self-feed with splints, able to drive
 C7 Manual wheelchair, independent transfer, dressing (with aids), feeding, bathing, self-care. Bladder and bowel
care with assistance
 C8–T4 Independent with most activities of daily living, and bowel and bladder care
 T5–T12 As above but with more ease. Independent with all self-care
 L1–L5 Independent. Walk with short or long leg braces
 S1–S5 Independent, able to walk if able to push off (S1) (may need brace). Bladder, bowel and sexual function
may remain compromised

22. Most common cause of compartment syndrome is:


a. Third degree burn
b. Fracture
c. Soft tissue contusion
d. Tight casts

Ans: b
Ref: Bailey and Love’s Short Practice of Surgery 27th Edition, Page No: 408
 Compartment syndrome is raised pressure in an osseofascial compartment to a level that compromises tissue
perfusion.
 There are several causes of compartment syndrome, fractures being the most common (70%), followed by soft
tissue contusions (23%).
 Rarer causes include: bleeding disorders including anticoagulation, burns (particularly circumferential 3 rd degree
burns); post-ischaemic swelling (reperfusion injury); tight casts/dressings; and extravasation of intravenous
infusions (contrast under pressure).

23. Simmonds’ test helps to check integrity of the following tendon?


a. Brachioradialis
b. Flexor hallucis longus
c. Achilles
d. Quadriceps

Ans: c
Ref: Bailey and Love’s Short Practice of Surgery 27th Edition, Page No: 460
Achilles tendon test
 Feel the gastrocnemius and soleus bellies and the whole length of the tendon for gaps (rupture), tenderness or
swelling.
 Also identify the posterolateral (Haglund’s) prominence of the calcaneus and palpate the retro-Achilles bursa.
 The test for integrity of the tendon is the Thompson’s or Simmonds’ test. Do not be misled by the patient’s ability
to stand on tiptoes – some people can do this using their long toe flexors alone. Lie the patient prone and allow
their calves to rest on your forearms. Squeeze each calf in turn and watch for movement at the ankle joint. Lack of
movement may indicate a rupture.

24. A 65-year-old woman is hospitalized with gallstone pancreatitis and noted to have significant abdominal pain,
emesis, tachycardia, and tachypnea. Her amylase level is 3100 IU/L, glucose is 120 mg/dL, and calcium level is 13
mg/dL. Which of the following is most likely to correlate with poor prognosis in disease severity?
a. The patient’s age
b. The high amylase level
c. A glucose level less than 140 mg/dL
d. Hypercalcemia

Ans: a

Paradise Institute & Self Study Centre Page 8


PARADISE WEEKLY MODEL TEST CEE MD/MS BASED MAGH 16

Ref: Bailey and Love’s Short Practice of Surgery 27th Edition


The level of the amylase or lipase does not correlate with disease severity. Hypoxemia, hypocalcemia, and age more than
55 years are some of the poor prognostic factors based on Ranson criteria. High body mass index has been reported to be
associated with worse prognosis related to acute pancreatitis.

25. Which of the following cleft type is least common?


a. Cleft lip alone
b. Cleft lip and palate
c. Isolated cleft palate
d. Any

Ans: a
Ref: Bailey and Love’s Short Practice of Surgery 27th Edition, Page No: 688
The incidence of cleft lip and palate is 1:600 live births and of isolated cleft palate is 1:1000 live births.
Although cleft lip and palate is an extremely diverse and variable congenital abnormality, several distinct sub-groups exist,
namely cleft lip with/without cleft palate (CL/P), cleft palate (CP) alone and submucous cleft palate (SMCP).
The typical distribution of cleft types is:
 Cleft lip alone: 15%
 Cleft lip and palate: 45%
 Isolated cleft palate: 40%

26. A 55-year-old man involved in an automobile accident is unresponsive and is intubated at the scene. On arrival
in the emergency department, he responds to painful stimulation. His systolic BP is 60 mm Hg, his HR is 140 bpm,
his neck veins are distended, and his breath sounds are absent on the left side. Immediate management should
involve which of the following?
a. Insertion of a central venous line on the right side
b. Insertion of an 18-gauge needle in the left second intercostal space
c. Pericardiocentesis
d. CT scan of head

Ans: b
• The patient has tension pneumothorax, as evidenced by distended neck veins and absent breath sounds. Increased
intrathoracic pressure interferes with venous return to the heart, resulting in shock. Immediate management should be
insertion of a large-bore needle in the left second intercostal space, followed by insertion of a chest tube. In a trauma
patient, venous access should be achieved by inserting two large-bore (16-gauge) angiocatheters in the cubital veins.
• Insertion of a central venous line on the right side should not be done, because it carries the risk of producing
pneumothorax in the opposite side.

27. A 32-year-old woman is noted to have nipple discharge. She is concerned about the possible association with
breast cancer. Which of the following etiologies of nipple discharge most likely increases the risk of breast cancer?
a. Fibrocysticchanges
b. Blood-tingeddischarge
c. Intraductalpapilloma
d. Diffusepapillomatosis

Ans: d
Ref: Bailey and Love’s Short Practice of Surgery 27th Edition
Diffuse papillomatosis increases the risk of cancer.

28. Bentall’s operation is done in:


a. Venacava
b. Aorta
c. Peripheral artery
d. Thoracic duct

Paradise Institute & Self Study Centre Page 9


PARADISE WEEKLY MODEL TEST CEE MD/MS BASED MAGH 16

Ans: b
Ref: Bailey and Love’s Short Practice of Surgery 27th Edition, Page No: 909
The coronary ostia require reimplantation into the graft (Bentall’s operation). Bentall procedure is performed for the repair
of ascending aortic root lesions. Typically the native aortic root and aortic valve are replaced with a composite graft that
comprises both ascending aortic and aortic valve grafts, to which the coronary arteries are anastomosed.

29. A 35-year-old teacher presents with mildly swollen calf with paresthesia. On USG, there was incompetence of
Saphenofemoral junction with few dilates and tortuous veins in lower leg. Diagnosis is:
a. Deep vein thrombosis
b. Varicose vein
c. Muscle strain
d. Venous ulcer

Ans: b
Ref: Bailey and Love’s Short Practice of Surgery 27th Edition, Page No: 975
Varicose vein
 The presence of tortuous dilated subcutaneous veins is usually clinically obvious. These are confined to the GSV
and SSV systems in approximately 60% and 20% of cases, respectively.
 The distribution of varicosities may indicate which superficial axis is defective; medial thigh and calf varicosities
suggest GSV incompetence, posterolateral calf varicosities are suggestive of SSV incompetence, whereas
anterolateral thigh and calf varicosities may indicate isolated incompetence of the ASV. Any of the clinical
features above may be present. Large dilated veins around the SFJ may present as a (usually painless) lump,
emergent when standing and disappearing when recumbent. This is a saphena varix.
 Gentle palpation over the varix during coughing may elicit a thrill, though it may be mistaken for a groin hernia.

30. A 50-year-old man is noted to have painless jaundice that on CT scan reveals a periampullary tumor. Which of
the following is the most common periampullary cancer?
a. Pancreaticadenocarcinoma
b. Cholangiocarcinoma
c. Ampullarycarcinoma
d. Duodenaladenocarcinoma

Ans: a
Ref: SRB's Manual of Surgery; 5th Edition
Pancreatic adenocarcinoma is the most common of the periampullary tumors. These tumors can give rise to painless
jaundice, which is a common sign of pancreatic cancer, specifically the head of the pancreas..

31. Green discharge from breast is most commonly associated with:


a. Duct papilloma
b. Duct ectasia
c. Retention cyst
d. Fibroadenosis

Ans: b
Ref: Bailey and Love’s Short Practice of Surgery 27th Edition, Page No: 864
• Duct ectasia of the breast or mammary duct ectasia or plasma cell mastitis is a condition in which the lactiferous
duct becomes blocked or clogged. This is the most common cause of greenish discharge. Mammary duct ectasia
can mimic breast cancer. It is a disorder of peri- or post-menopausal age.
• Duct ectasia syndrome is a synonym for non-puerperal mastitis but the term has also been occasionally used to
describe special cases of fibrocystic diseases, mastalgia or as a wastebasket definition of benign breast disease.

32. Sebaceous cyst does not occur in:


a. Scalp

Paradise Institute & Self Study Centre Page 10


PARADISE WEEKLY MODEL TEST CEE MD/MS BASED MAGH 16

b. Scrotum
c. Back
d. Sole

Ans: d
Ref: SRB's Manual of Surgery; 4th Edition, Page No: 79
Sebaceous cyst:
 The scalp, ears, back, face, and upper arm, are common sites of sebaceous cysts, though they may occur anywhere
on the body except the palms of the hands and soles of the feet. In males a common place for them to develop is
the scrotum and chest.
 They are more common in hairier areas, where in cases of long duration they could result in hair loss on the skin
surface immediately above the cyst. They are smooth to the touch, vary in size, and are generally round in shape.
They are generally mobile masses that can consist of:
• Fibrous tissues and fluids,
• A fatty (keratinous) substance that resembles cottage cheese, in which case the cyst may be called "keratin cyst".
This material has a characteristic "cheesy" or foot odor smell,
• A somewhat viscous, serosanguineous fluid (containing purulent and bloody material).

33. Flower vase pattern of ureter is characteristic of:


a. Sigmoid kidney
b. Horseshoe kidney
c. Crossed ectopia
d. L-shaped kidney

Ans: b
Ref: SRB's Manual of Surgery; 4th Edition, Page No: 1068
Horse Shoe Kidney
• Both kidneys are fused at lower pole.
• M>F
• Ascent of horse shoe kidney is prevented by inferior mesenteric artery so kidney is confined at L4 level.
• Most common association is VUR. PUJ obstruction, hypospadias, undescended testes, bicornuate uterus, trisomy
18, Turner may also be seen.
• Flower vase appearance in nephrogram
• Complication: Renal calculi

34. Which of the following is the most common metabolic complication of


primary hyperparathyroidism?
a. Kidneystones
b. Osteoporosis
c. Pancreatitis
d. Gout

Ans: a
Ref: Bailey and Love’s Short Practice of Surgery 27th Edition
Kidney stones are the most common metabolic complications associated with hyperparathyroidism, occurring in 15% to
20% of patients with the disease.

35.In which segment of the intestine is ligation of the inferior mesenteric artery during aortic aneurysm repair most
likely to produce ischemia?
a. Transversecolon
b. Splenic flexure colon
c. Rightcolon
d. Distal small bowel

Paradise Institute & Self Study Centre Page 11


PARADISE WEEKLY MODEL TEST CEE MD/MS BASED MAGH 16

Ans: b
Ref: Bailey and Love’s Short Practice of Surgery 27th Edition
Splenic flexure colon. The SMA supplies the bowel from distal duodenum to the midtransverse colon. The IMA supplies
the descending colon down to the upper rectum. The lower rectum receives blood supply from the hemor- rhoidal arteries
that originate from the iliac artery. The “water-shed” areas (areas between defined blood supplies that rely on blood flow
from collateral circulations) of the GI tract include the splenic flexure of colon and distal sigmoid colon/upper rectum.

36.For the diagnosis of acute appendicitis, 2 points in MANTRELS is given for the sign:
a. RIF pain
b. RIF tenderness
c. Nausea
d. WBC 46,000

Ans: b
Ref: Bailey and Love’s Short Practice of Surgery 27th Edition, Page No: 1307
The Alvarado (MANTRELS) score for diagnosis of acute appendicitis:
Score

Symptoms Migratory RIF pain 1


Anorexia 1
Nausea and vomiting 1

Signs Tenderness (RIF) 2


Rebound tenderness 1
Elevated temperature 1

Lab Leukocytosis 2
Shift to left 1

Total 10

The most widely used clinical and laboratory based is the Alvarado score. A score of 7 or more is strongly predictive of
acute appendicitis.
In patients with an equivocal score (5–6), abdominal ultrasound or contrast-enhanced CT examination further reduces the
rate of negative appendicectomy.

Obstetrics and Gynecology

37. A woman presents with sudden onset of profuse vaginal bleeding one week after a normal vaginal delivery. She
complains of an offensive vaginal discharge prior to this episode. What is the most likely cause?
a. Endometritis
b. Low platelet count
c. Poor healing of the episiotomy wound
d. Retained placental tissue

Ans: a
Retained products and infection are the commonest causes of secondary postpartum haemorrhage.
In most cases both causes are present. If retained placental parts were present, continuous mild bleeding will be present
before this episode.
Since she had an offensive vaginal discharge before the onset of bleeding, endometritis is the most likely cause.

Paradise Institute & Self Study Centre Page 12


PARADISE WEEKLY MODEL TEST CEE MD/MS BASED MAGH 16

However, an ultrasound scan should be done to exclude retained placental tissue, in all cases of secondary postpartum
haemorrhage.

38. A woman complains of sudden onset of moderately severe bleeding without abdominal pain at 31 weeks. She is
pale, the blood pressure is 80/60 mmHg and the pulse rate is 130 beats per minute. Bleeding continues after
admission to the ward. The uterus is soft and non-tender. She has a single fetus in cephalic presentation. The head is
high. The fetal heart rate is 140 beats per minute. What is the first step management?
a. Admit to the intensive care unit and keep under observation
b. Deliver by lower segment caesarean section immediately
c. Perform a speculum examination to exclude co-existing incidental causes
d. Transfuse 3 pints of blood rapidly

Ans: d
 The first step in the management of obstetric haemorrhage is resuscitation.
 The most important aspect of resuscitation is replacement of the circulating volume with blood transfusion.

39. Glands of littre are homologous to:


a. Bartholin gland
b. Cowper’s gland
c. Skene glands
d. Glands on labia

Ans: d
 The glands of labia minora are homologous to the glands of littre (glandulaepreputiales) of the penile portion of the
male urethra.
 Periurethral (Skene) glands (homologous to the prostate gland) open into the distal portion
 Minor vestibular glands (homologous to glands of Littré in males) open along the entire length

40. Vaginal defence is lost:


a. Within 10 days of birth
b. After 10 days of birth
c. During pregnancy
d. At puberty

Ans: b
Ref: Dutta Gynecology 6th Edition, Page No: 126
Vaginal defence

Paradise Institute & Self Study Centre Page 13


PARADISE WEEKLY MODEL TEST CEE MD/MS BASED MAGH 16

Anatomic: (i) Apposition of the anterior and posterior walls with its transverse rugae; (ii) Stratified epithelium devoid of
glands.
Physiologic: This is maintained by the hormone oestrogen.
 At Birth, under the influence of maternal estrogen circulating into the newborn, the vaginal epithelium becomes
multilayered. The desquamated epithelium containing glycogen is converted into lactic acid probably by enzymatic
action for the first 48 hours.
 Subsequently, the Doderlein’s bacilli appear probably from the gut and convert the glycogen into lactic acid.
 As a result, for about 10-12 days following birth, the vaginal defence is good and infection is unlikely.
 Thereafter and upto puberty, there is no circulatory estrogen. The vaginal epithelium is reduced to few layers;
glycogen is absent and so also the Doderlein’s bacillus. The vaginal pH becomes neutral or alkaline.
 During the reproductive period with high oestrogen, the vaginal defence is fully restored. But again in
postmenopause, after the withdrawal of estrogen, the vaginal defence is lost.
 It should be emphasized that only the Doderlein’s bacilli can grow in the acidic media with pH 4–4.5.
 But when the pH increases, the other organisms normally present in the vagina will grow.

41. Which of the following hormone is raised in polycystic ovarian syndrome?


a. 17 – OH progesterone
b. Follicular stimulating hormone
c. Luteinizing hormone
d. Thyroid stimulating hormone

Ans: c
Ref: Dutta Gynecology 6th Edition, Page No: 460
Serum values in PCOS:
 LH level is elevated and/or the ratio LH: FSH is > 2:1.
 Raised level of estradiol and estrone: The estrone level is markedly elevated.
 SHBG level is reduced.
 Hyperandrogenism—mainly from the ovary but less from the adrenals. Androstenedione is raised.
 Raised serum testosterone (> 150 ng/dl) and DHEA–S may be marginally elevated.
 Insulin Resistance (IR): Raised fasting insulin levels > 25 μIU/ml and fasting glucose/insulin ratio < 4.5 suggests
IR (50%). Levels of serum insulin response > 300 μIU/ml at 2 hours postglucose (75 gm) load, suggests severe IR.

42. Vaginal epithelium is derived from:


a. Endoderm of urogenital sinus
b. Mesoderm of urogenital sinus
c. Endoderm of genital ridge
d. Mesoderm of genital ridge

Ans: a
Ref: Dutta Gynecology 6th Edition, Page No: 37
 Vagina is developed mainly from the Müllerian ducts and partly from the urogenital sinus.
 Upper three-fifth above the hymen develop from the fused uterovaginal canal of the Müllerian ducts.
 Mucous membrane is developed from the endoderm of the canalized (vaginal plate) sinovaginal bulb (urogenital
sinus).
 The musculature is developed from the mesoderm of the fused caudal vertical part of the Müllerian ducts.
 The hymen is developed from the junction of the Müllerian tubercle (mesodermal) and the urogenital sinus
(endodermal).
 Lower one-fifth below the hymen is developed entirely from the endoderm of the urogenital sinus.
 Vaginal introitus is developed from the ectoderm of the genital folds after rupture of the bilaminar urogenital
membrane.

43. Most useful investigation for VVF is:


a. Three swab test

Paradise Institute & Self Study Centre Page 14


PARADISE WEEKLY MODEL TEST CEE MD/MS BASED MAGH 16

b. Cystoscopy
c. Urine culture
d. IVP

Ans: b
Ref: Dutta Gynecology 6th Edition, Page No: 417, 418
Vesicovaginal Fistula (VVF)
There is communication between the bladder and the vagina and the urine escapes into the vagina causing true
incontinence. This is the commonest type of genitourinary fistula.
Diagnosis:
Big fistula
• Visible fistula tract
• Obvious escape of urine
Tiny fistula
• Dye test
• In knee-chest position escape of bubbles of air when the patient coughs
• Three-swab test
Confusion in diagnosis
• Cystoscopy
Three swab test:
• Upper most swab soaked with urine but unstained with dye. The lower two fistula swabs remain dry:
Ureterovaginal fistula
• Upper and lower swabs remain dry but the middle swab stained with dye: Vesicovaginal fistula
• The upper two swabs remain dry but the lower swab stained with dye: Urethrovaginal fistula

44. Composition of Nova - T:


a. Copper and silver
b. Copper and aluminium
c. Copper only
d. Copper and molybdenum

Ans: a
Ref: Shaw Gynecology 16th Edition, Page No: 269
Copper-carrying devices:
• In these, copper wire of surface area 200 to 250 mm is wrapped round the vertical stem of a polypropylene frame.
• Among these devices are Copper T 200, Copper 7, Multiload Copper 250, Copper T 380, Copper T 220 and Nova
T.
• The copper devices are more expensive than inert devices but are reported to exert a better contraceptive effect,
with fewer side effects. They have an effective life of about 3–5 years. It is estimated that about 50 mcg of copper
is eluted daily in the uterus.
• Copper T 380A, known as Paraguard, has a lifespan of 10 years.
• Nova T has silver added to the copper wire, thereby increasing its lifespan to 5 years.

45. Calcareous degeneration occurs most commonly in which type of fibroids:


a. Submucous
b. Subserous
c. Interstitial
d. Cervical

Ans: b
Ref: Shaw Gynecology 16th Edition, Page No: 395 | Ref: Dutta Gynecology 6th Edition, Page No: 275
Calcareous Degeneration:
• In calcareous degeneration, phosphates and carbonates of lime are deposited in the periphery along the course of
the vessels.

Paradise Institute & Self Study Centre Page 15


PARADISE WEEKLY MODEL TEST CEE MD/MS BASED MAGH 16

• The best examples of calcareous myomas are those in old patients with long-standing myomas.
• They are like ‘womb-stones’ in graveyards.
• Calcareous tumours are easily identified by radiography.
• Calcific degeneration (10%) usually involves the subserous fibroids with small pedicle or myomas of
postmenopausal women. It is usually preceded by fatty degeneration. There is precipitation of calcium carbonate or
phosphate within the tumor. When whole of the tumor is converted into a calcified mass, it is called “womb stone”.

46. Critical obstetric conjugate for trial of labour is:


a. 8.5 cm
b. 9.0 cm
c. 9.5 cm
d. 10.0 cm

Ans: d
Ref: Dutta Obstetrics 8th Edition, Page No: 100
Obstetric conjugate:
• It is the distance between the midpoint of the sacral promontory to prominent bony projection in the midline on the
inner surface of the symphysis pubis.
• The point is somewhat below its upper border.
• It is the shortest anteroposterior diameter in the anteroposterior plane of the inlet.
• It measures 10 cm (4"). It cannot be clinically estimated but is to be inferred from the diagonal conjugate—1.5–2
cm (¾") to be deducted or by lateral radiopelvimetry.

47. Best test for estimating hCG:


a. Radioimmunoassay
b. ELISA
c. Radioreceptor assay
d. Bioassay

Ans: a
Ref: Dutta Obstetrics 8th Edition, Page No: 228
Detection of hCG in urine or serum in follow up of molar pregnancy
Urine or serum assays are carried out at every visit. Initially, the less sensitive and less costly immunological test may be
carried out until the test becomes negative. Thereafter, it is preferable to use a more sensitive serum hCG level by
radioimmunoassay.

48. Which does not cross placenta?


a. Heparin
b. Morphine
c. Naloxone
d. Warfarin

Ans: a
Most drugs with MW < 500 Da cross the placenta, and most drugs with MW > 1000 Da do not cross the placenta (ex.
heparin, protamine, insulin).

Pediatrics

49. A 6-year-old boy presents with a 3-day history of fever up to 102.2°F and rash. The mother reports associated
cough, drainage from the right eye, and nasal irritation. What is the most likely diagnosis?
a. Rubeola
b. Roseola
c. Hand, foot, and mouth disease
d. Rubella
Paradise Institute & Self Study Centre Page 16
PARADISE WEEKLY MODEL TEST CEE MD/MS BASED MAGH 16

Ans: a
Ref: Nelson Textbook of Pediatrics, Elsevier, 20th Edition, Page No: 2714
The boy symptoms are consistent with rubeola (measles). He displays the classic rash and “three Cs” (cough,
conjunctivitis, and coryza).

50. The most common cause of Cushing syndrome is:


a. Prolonged exogenous administration of glucocorticoid hormones
b. 3β-Hydroxysteroid dehydrogenase defect
c. Bilateral adrenal hyperplasia
d. McCune-Albright syndrome

Ans: a
Ref: Nelson Textbook of Pediatrics, Elsevier, 20th Edition, Page No: 2723
Cushing syndrome is the result of abnormally high blood levels of cortisol or other glucocorticoids. This can be iatrogenic
or the result of endogenous cortisol secretion, a result of either an adrenal tumor or of hypersecretion of corticotropin
(adrenocorticotropic hormone [ACTH]) by the pituitary (Cushing disease) or by a tumor.
The most common cause of Cushing syndrome is prolonged exogenous administration of glucocorticoid hormones,
especially at the high doses used to treat lymphoproliferative disorders. This rarely represents a diagnostic challenge, but
management of hyperglycemia, hypertension, weight gain, linear growth retardation, and osteoporosis often complicates
therapy with corticosteroids.

51. A 2-year-old girl is brought to the ED by her parents for fever up to 103°F, decreased oral intake, and “not
acting herself.” On examination, the child is lethargic. She has pain with flexion of her neck. Note small,
nonblanching red dots on her legs and torso. What is the best management option?
a. PO challenge and reassess
b. Begin IV hydration and empiric antibiotic treatment
c. Fever control and discharge home
d. Obtain laboratory testing to narrow the differential diagnosis

Ans: b
Ref: Nelson Textbook of Pediatrics, Elsevier, 20th Edition
This girl is lethargic. She has fever, meningeal signs, and a rash concerning for meningococcemia. Empiric antibiotic
therapy should be started immediately.

52. In early childhood, trachoma begins as:


a. Follicular conjunctivitis
b. Vascular pannus
c. Follicular keratitis
d. Uveitis

Ans: a
Ref: Nelson Textbook of Pediatrics, Elsevier, 20th Edition, Page No: 1493
Trachoma
• Trachoma is the most important preventable cause of blindness in the world.
• It is caused primarily by the A, B, Ba, and C serotypes of C. trachomatis.
• Flies are a common vector.
• Trachoma begins as a follicular conjunctivitis, usually in early childhood.
• The follicles heal, leading to conjunctival scarring that can result in an entropion, with the eyelid turning inward so
that the lashes abrade the cornea. It is the corneal ulceration secondary to the constant trauma that leads to scarring
and blindness.
• Bacterial superinfection can also contribute to scarring.
• Blindness occurs years after the active disease.
• Trachoma can be diagnosed clinically.

Paradise Institute & Self Study Centre Page 17


PARADISE WEEKLY MODEL TEST CEE MD/MS BASED MAGH 16

• The World Health Organization suggests that at least 2 of 4 criteria must be present for a diagnosis of trachoma:
lymphoid follicles on the upper tarsal conjunctivae, typical conjunctival scarring, vascular pannus, and limbal
follicles.
• The diagnosis is confirmed by culture or staining tests for C. trachomatis performed during the active stage of
disease. Serologic tests are not helpful clinically because of the long duration of the disease and the high
seroprevalence in endemic populations.

53. A 15-month-old girl is playing quietly in your waiting room. The skin around her mouth is faintly blue, but she
appears comfortable. She arises from her squatting position to run after her brother, and she suddenly becomes
dyspneic and cyanotic. She returns to a squatting position and soon is breathing comfortably with only slight
perioral cyanosis. Which of the following would you expect to see on her chest radiograph?
a. A “boot-shaped” heart
b. An“eggonastring”
c. Lung hyperinflation
d. Pulmonarycongestion

Ans: a
Ref: Rudolph’s Pediatrics. 21st ed. New York, NY: McGraw-Hill; 2003: 1814-1826.
This child has TOF; she experiences improvement when squatting and “tet” (hypercyanotic) spells when running. The
“boot-shaped” heart is a characteristic chest radiographic finding.

54. Icterus Neonatorum is:


a. Indirect bilirubin in umbilical cord serum 1-3 mg/dL
b. Jaundice becomes visible on the 2nd or 3rd day
c. Physiological jaundice
d. All of the above

Ans: d
Ref: Nelson Textbook of Pediatrics, Elsevier, 20th Edition, Page No: 873, 874
Physiologic jaundice (Icterus Neonatorum)
Under normal circumstances, the level of indirect bilirubin in umbilical cord serum is 1-3 mg/dL and rises at a rate of <5
mg/dL/24 hr; thus, jaundice becomes visible on the 2nd or 3rd day, usually peaking between the 2nd and 4th days at 5-6

Paradise Institute & Self Study Centre Page 18


PARADISE WEEKLY MODEL TEST CEE MD/MS BASED MAGH 16

mg/dL and decreasing to <2 mg/dL between the 5th and 7th days after birth. Jaundice associated with these changes is
designated physiologic and is believed to be the result of increased bilirubin production from the breakdown of fetal red
blood cells combined with transient limitation in the conjugation of bilirubin by the immature neonatal liver.

55. Most common life-threatening emergency of the gastrointestinal tract in the newborn period is:
a. Diarrhoea
b. Duodenal atresia
c. Necrotizing enterocolitis
d. Hypertrophic pyloric stenosis

Ans: c
Ref: Nelson Textbook of Pediatrics, Elsevier, 20th Edition, Page No: 869
Necrotizing Enterocolitis
• NEC is the most common life-threatening emergency of the gastrointestinal tract in the newborn period.
• The disease is characterized by various degrees of mucosal or transmural necrosis of the intestine.
• The cause of NEC remains unclear but is most likely multifactorial.
• The incidence of NEC is 1-5% of infants in neonatal ICUs.
• Both incidence and case fatality rates increase with decreasing birth weight and gestational age.
• Because very small, ill preterm infants are particularly susceptible to NEC, a rising incidence may reflect improved
survival of this high-risk group of patients.
• Infants with NEC have a variety of signs and symptoms and may have an insidious or sudden catastrophic onset.
• The onset of NEC is usually in the 2nd or 3rd wk of life but can be as late as 3 marginal osteophytes in VLBW
infants.
• Age of onset is inversely related to gestational age.
• The first signs of impending disease may be nonspecific, including lethargy and temperature instability, or related
to gastrointestinal pathology, such as abdominal distention and gastric retention.

56. Foramen of Morgagni Hernia occurs through which diaphragmatic defect?


a. Anteromedial diaphragmatic defect
b. Posterolateral diaphragmatic defect
c. Posteromedial diaphragmatic defect
d. Anterolateral diaphragmatic defect

Ans: a
Ref: Nelson Textbook of Pediatrics, Elsevier, 20th Edition, Page No: 864
Foramen of Morgagni Hernia
• The anteromedial diaphragmatic defect through the foramen of Morgagni accounts for 2-6% of diaphragmatic
hernias.
• Failure of the sternal and crural portions of the diaphragm to meet and fuse produces this defect. These defects are
usually small, with a greater transverse than anteroposterior diameter, and are more commonly right-sided (90%)
but may be bilateral. The transverse colon or small intestine or liver is usually contained in the hernial sac.
• The majorities of children with these defects are asymptomatic and are diagnosed beyond the neonatal period. The
diagnosis is usually made on chest radiograph when a child is evaluated for another reason. The anteroposterior
radiograph shows a structure behind the heart, and a lateral film localizes the mass to the retrosternal area. Chest
CT or MRI will confirm the diagnosis.
• When symptoms occur, they can be recurrent respiratory infections, cough, vomiting, or reflux; in rare instances,
incarceration may occur.
• Repair is recommended for all patients, in view of the risk of bowel strangulation, and can be accomplished
laparoscopically or by an open approach. Prosthetic material is rarely required.

A helpful mnemonic for remembering the features of a Bochdalek hernia is:


 5 Bs
Mnemonic
 B: Bochdalek

Paradise Institute & Self Study Centre Page 19


PARADISE WEEKLY MODEL TEST CEE MD/MS BASED MAGH 16

 B: big
 B: back and lateral, usually on the left side
 B: baby
 B: bad (associated with pulmonary hypoplasia)
To remember the side in which a Bochdalek hernia more commonly occurs (and to contrast that with Morgagni hernias)
Bochdalek has an L for left and Morgagni has an R for right.

57. A 16-year-old adolescent male complains of intermittent cola-colored urine of several years’ duration, usually
when he has a “cold.” He is otherwise well and has no medical complaints. When the dark- colored urine is present,
he has no dysuria. None of his family members has similar complaints or renal disease. On physical examination he
is normotensive and appears healthy. Which of the following is the most likely because of his intermittent
hematuria?
a. Acute poststreptococcal glomerulonephritis
b. Henoch-Schönleinpurpuranephritis
c. IgA nephropathy
d. Rapidly progressive glomerulonephritis

Ans: c
Ref: Oski’s Pediatrics: Principles and Practice. 4th ed. Philadelphia
Recurrent painless gross hematuria, frequently associated with an upper respiratory tract infection, is typical of IgA
nephropathy. These patients may develop chronic renal disease over decades. If protein- uria, hypertension, or impaired
renal function were found, a biopsy would be necessary. The other options are not consistent with the asymptomatic,
intermittent nature of this patient’s problem.

58. Most important risk factor for neonatal respiratory distress syndrome-
a. Diabetic mother
b. Asphyxia
c. Prematurity
d. Twin pregnancy

Ans: c
Ref: OP Ghai Pediatrics 9th Edition, Page No: 168
Table: Pulmonary causes of respiratory distress

59. A 3650-g term infant has ambiguous genitalia, including an enlarged clitoris/microphallus and one palpable
testis in the labioscrotal folds. Sonogram reveals a uterus and ovaries. Which of the following is the most likely
explanation for the child’s ambiguous genitalia?
a. Congenitaladrenal hyperplasia
b. Female pseudohermaphroditism
c. Male pseudohermaphroditism
d. Truehermaphroditism

Ans: d
Ref: Rudolph’s Pediatrics. 21st ed. New York, NY: McGraw-Hill; 2003:2079-2101.
The gonad in the labioscrotal fold suggests a testis, but a uterus and an ovary on sonography are highly suggestive of a true
hermaphrodite. Gender assignment in this case should be based on the possibility of surgical correction of the external
genitalia. Assignment of female sex and an attempt to preserve ovarian tissue is appropriate.
Paradise Institute & Self Study Centre Page 20
PARADISE WEEKLY MODEL TEST CEE MD/MS BASED MAGH 16

60.True about Measles rash appearance:


a. Along with Koplik spots
b. 1-2 days before Koplik spots
c. 1-2 days after Koplik spots
d. Post-measles stage

Ans: c
Ref: OP Ghai Pediatrics 8th Edition, Page No: 213
• Starts 12-24 hours after appearance of Koplik spots
• Retro-auricular origin spreads to other parts of body (fades in opposite direction)
• Takes 1-2 days to cover body
• Fever highest and patient sickest on day of rash

ENT

61. Chronic rhinosinusitis is termed when duration is:


a. ≥ 3 weeks
b. ≥ 6 weeks
c. ≥ 12 weeks
d. ≥ 20 weeks

Ans: c
Ref: PL Dhingra and Shruti Dhingra ENT and HNS 7th Edition, Page No: 213
• Acute RS: Symptoms lasting for less than 4 weeks with complete resolution.
• Subacute RS: Duration 4-12 weeks.
• Chronic RS: Duration ≥ 12 weeks.
• Recurrent RS: Four or more episodes of RS per year; each lasting for 7-10 days or more with complete resolution
in between the episodes.

62. Where are otoacoustic emissions generated from?


a. Inner hair cells
b. Outer hair cells
c. Tympanic membrane
d. VIIIth nerve

Ans: b
Ref: PL Dhingra and Shruti Dhingra ENT and HNS 7th Edition, Page No: 132
 Otoacoustic emissions (OAEs) are generated at outer hair cells and can be picked up from the external ear as the
energy produced by them travels in reverse direction from outer hair cells → ossicles → tympanic membrane →
ear canal where it is picked up.
 OAEs are absent if outer hair cells in the cochlea are nonfunctional or there is middle ear effusion or canal debris
due to meconium which may persist for 3–4 days. They are normal even when VIIIth nerve is nonfunctional. Thus
can be used in the diagnosis of neuropathy of VIIIth nerve.

63. Which segment of the intratemporal facial nerve is the shortest and narrowest?
a. Labyrinthine segment
b. Meatal segment
c. Horizontal segment
d. Vertical segment

Ans: a
Ref: PL Dhingra and Shruti Dhingra ENT and HNS 7th Edition, Page No: 99
Intratemporal Part. From internal acoustic meatus to stylomastoid foramen. It is further divided into:
Paradise Institute & Self Study Centre Page 21
PARADISE WEEKLY MODEL TEST CEE MD/MS BASED MAGH 16

 Meatal segment (8–10 mm). Within internal acoustic meatus.


 Labyrinthine segment (4.0 mm). From fundus of meatus to the geniculate ganglion where nerve takes a turn
posteriorly forming a “genu.” The nerve in the labyrinthine segment has the narrowest diameter (0.61–0.68 mm)
and the bony canal in this segment is also the narrowest. Thus oedema or inflammation can easily compress the
nerve and cause paralysis. This is also the shortest segment of the nerve.
 Tympanic or horizontal segment (11.0 mm). From geniculate ganglion to just above the pyramidal eminence. It
lies above the oval window and below the lateral semicircular canal.
 Mastoid or vertical segment (13.0 mm). From the pyramid to stylomastoid foramen. Between the tympanic and
mastoid segments is the second genu of the nerve.

64. What is false about crocodile tears?


a. Bilateral lacrimation with mastication
b. Faulty regeneration of parasympathetic fibres following paralysis of facial nerve
c. Lacrimal glands get supplied by branches of facial nerve
d. Treatment with tympanic neurectomy

Ans: a
Ref: PL Dhingra and Shruti Dhingra ENT and HNS 7th Edition, Page No: 108
Crocodile Tears (Gustatory Lacrimation). There is unilateral lacrimation with mastication. This is due to faulty
regeneration of parasympathetic fibres which now supply lacrimal gland instead of the salivary glands. It can be treated by
section of greater superficial petrosal nerve or tympanic neurectomy.

65. Which of the following is hypermotility disorder of esophagus?


a. Diffuse oesophagealspasm
b. Gastroesophageal reflux
c. Scleroderma
d. Amyotrophic lateral sclerosis

Ans: a
Ref: PL Dhingra and Shruti Dhingra ENT and HNS 7th Edition, Page No: 389
Motility disorders of oesophagus
They can be divided into:
• Hypermotility disorder, e.g. cricopharyngeal spasm, diffuse oesophageal spasm, nut crackeroesophagus.
• Hypomotility disorders, e.g. cardiac achalasia, gastroesophageal reflux, scleroderma, amyotrophic lateral sclerosis.

66. Commonest cause of Acute suppurative otitis media in children is


a. H. influenzae
b. S. pneumoniae
c. S. aureus
d. Pseudomonas

Ans: b
Ref: Current otolaryngology 2nd edition, Page No: 656
Most common cause of ASOM
 Streptococcus pneumoniae (35 -40 %)
 H. Influenza 25-30%
 M. Catarrhalis 10-20%

67. Myringoplasty is also known as which type of tympanoplasty?


a. Type I
b. Type II
c. Type III
d. Type IV

Paradise Institute & Self Study Centre Page 22


PARADISE WEEKLY MODEL TEST CEE MD/MS BASED MAGH 16

Ans: a
Ref: Dhingra ENT, 7th edition, page 32
Types of Tympanoplasty. Wullstein classified tympanoplasty into five types
Type I Defect is perforation of tympanic membrane which is repaired with a graft. It is also called myringoplasty.

Type II Defect is perforation of tympanic membrane with erosion of malleus. Graft is placed on the incus or
remnant of malleus.

Type III Malleus and incus are absent. Graft is placed directly on the stapes head. It is also called
myringostapediopexy or columella tympanoplasty.
Type IV Only the footplate of stapes is present. It is exposed to the external ear, and graft is placed between the
oval and round windows. A narrow middle ear (cavum minor) is thus created to have an air pocket around
the round window. A mucosa-lined space extends from the eustachian tube to the round window. Sound
waves in this case act directly on the footplate while the round window has been shielded.

Type V Stapes footplate is fixed hut round window is functioning. In such cases, another window is created on
horizontal semicircular canal and covered with a graft. Also called fenestration operation.

68. Most common benign neoplasm of esophagus is:


a. Leiomyoma
b. Polyp
c. Lipoma
d. Hemangioma

Ans: a
Ref: PL Dhingra and Shruti Dhingra ENT and HNS 7th Edition, Page No: 390
Benign neoplasms of esophagus
Benign neoplasms are rare compared to malignant ones.
Leiomyoma
 Most common and accounts for two-thirds of all the benign neoplasms. It arises from the smooth muscle and
grows in the wall of oesophagus.
 Dysphagia is produced when tumour exceeds the diameter of 5 cm. Barium swallow shows an ovoid filling defect.
 Endoscopy reveals a submucosal swelling.
 Biopsy should not be taken.
 Treatment is enucleation of the tumour by thoracotomy.
Mucosal polyps, lipomas, fibromas and haemangiomas are other benign tumours. They are often pedunculated and
present in the oesophageal lumen.
Endoscopic removal is avoided because of the danger of oesophageal perforation. Treatment is surgical excision by
oesophagotomy.

69. Which of the following is associated with inspiratory stridor?


a. Laryngomalacia
b. Subglottic stenosis
c. Tracheal stenosis
d. Vocal cord paralysis

Ans: A
Ref: Dhingra ENT, 7th edition, page 333
Stridor is noisy respiration produced by turbulent airflow through the narrowed air passages. It may be heard during
inspiration, expiration or both:
Inspiratory stridor is often produced in obstructive lesions of supraglottis or pharynx, e.g. laryngomalacia or
retropharyngeal abscess.

Paradise Institute & Self Study Centre Page 23


PARADISE WEEKLY MODEL TEST CEE MD/MS BASED MAGH 16

Expiratory stridor is produced in lesions of thoracic trachea, primary and secondary bronchi, e.g. bronchial foreign body,
and tracheal stenosis.
Biphasic stridor is seen in lesions of glottis, subglottis and cervical trachea, e.g. laryngeal papillomas, vocal cord paralysis
and subglottis stenosis.

70. What is not a sequelae of otitis media?


a. Learning disabilities
b. Otitic hydrocephalus
c. Sensorineural hearing loss
d. Tympanosclerosis

Ans: b
Ref: PL Dhingra and Shruti Dhingra ENT and HNS 7th Edition, Page No: 83 – 84
Sequelae of otitis media are the direct result of middle ear infection and should be differentiated from complications. They
include:
 Perforation of tympanic membrane
 Ossicular erosion
 Atelectasis and adhesive otitis media
 Tympanosclerosis
 Cholesteatoma formation
 Conductive hearing loss due to ossicular erosion or fixation
 Sensorineural hearing loss
 Speech impairment
 Learning disabilities
The last two are secondary to loss of hearing in the developmental phase of the infant or child.
Complications of otitis media are classified into two main groups:
A. Intratemporal (within the confines of temporal bone)
 Mastoiditis
 Petrositis
 Facial paralysis
 Labyrinthitis
B. Intracranial
 Extradural abscess
 Subdural abscess
 Meningitis
 Brain abscess
 Lateral sinus thrombophlebitis
 Otitic hydrocephalus.

Orthopedics

71. Which of the following approaches is most appropriate for the clearance of the C-spine in a 25-year-old man
who the driver of a car struck from behind? He is hemodynamically stable, nonintoxicated, and has a GCS of 15.
a. NEXUS criteria
b. Canadian C-spine rule
c. CT of the C-spine
d. Remove the collar because he does not have any pain

Ans: b
Ref: Schwartz’s Principles of Surgery. 9th ed. New York, NY: McGraw-Hill; 2010:1557- 1608.
The Canadian C-spine rule (CCR) is an approach developed for the clear- ance of C-spines in asymptomatic patients
following low mechanism events. The CCR has been compared to the NEXUS criteria and found to be more specific and
sensitive in clearance of the C-spine.

Paradise Institute & Self Study Centre Page 24


PARADISE WEEKLY MODEL TEST CEE MD/MS BASED MAGH 16

72. What is Thompson’s approach for forearm surgeries used for?


a. Fracture of distal radius
b. Fracture of proximal radius
c. Fracture of distal ulna
d. Fracture of proximal ulna

Ans: b
Ref: Ebnezar Orthopedics 4th Edition, Page No: 717
 Forearm DCP plating
 Indications
 Fracture both bones in adults or fracture radius shaft or ulna.
Approaches:
 Dorsal approach: This is for fracture ulna.
 Thompson’s approach: This is fracture of proximal radius.
 Anterior approach: This is for distal radius fractures.

73. Which of the following patient’s presentation is most compatible with the Brown-Sequard syndrome?
a. A 20-year-old man with absence of all motor/sensory functions in all extremities
b. A 20-year-old man with greater weakness in the upper extremities than the lower extremities
c. A 20-year-old man with complete motor paralysis, loss of vibratory sensation and proprioception on the ipsilateral
side, and contralateral loss of pain and temperature sensation.
d. A 20-year-old man with fracture/dislocation of C5-C6 and intact motor/ sensory functions throughout

Ans: c
Ref: Schwartz’s Principles of Surgery. 9th ed. New York, NY: McGraw-Hill; 2010:1609-1645.
The Brown-Sequard syndrome is caused by posterior spinal cord injury, characterized by paralysis, loss of vibratory
sensation and proprioception on the ipsilateral side, and loss of pain and temperature sensation on the contralateral side.
Patient described in A is compatible with complete cord injury. The patient described in B is compatible with central cord
injury. The patient in D appears to have vertebral fractures/dislocation without neurologic compromises. The patient in E
has a spinal cord injury without radiographic abnormality (SCI- WORA); SCIWORAs occur more commonly in children
than adults.

74. Which of the following statement about treatment of fracture in children is incorrect?
a. Angulation < 10° is acceptable
b. Overlap of the fracture site by 1 to 2 cm is acceptable
c. Rotation < 5° is acceptable
d. All are try

Ans: c
Ref: Ebnezar Orthopedics 4th Edition, Page No: 106
The general rules in treatment of fractures in children
 Angulation > 10° is unacceptable.
 Rotation will not be compensated and hence is not acceptable.
 Overlap of the fracture site by 1 to 2 cm is acceptable as there is overgrowth following a long bone fracture.
 Correction occurs at the average rate of 1° per month.

75. A 27-year-old woman with a 1-week history of progressive pain radiating from the lumbar spine down the back
of the leg presents to the ED. Her physical examination is normal except for complaints of back pain with
movement. Which of the following is the most appropriate imaging test?
a. No imaging is necessary; attempt conservative therapy.
b. Obtain plain films of the lumbar spine.
c. Perform MRI.
d. Perform CT

Paradise Institute & Self Study Centre Page 25


PARADISE WEEKLY MODEL TEST CEE MD/MS BASED MAGH 16

Ans: a
Ref: Ebnezar Orthopedics 4th Edition
No imaging is necessary. If the patient has no risk factors in the history and physical examination for serious disease other
than sciatica, treat conserva- tively and do not perform any diagnostic tests in the ED.

76. What is incorrect about intertrochanteric fracture of femur?


a. Associated with severe pain and swelling along with shortening of involved limb
b. Complicated by mal-union
c. Patient is unable to walk
d. Tenderness localized over Scarpa’s triangle

Ans: d
Ref: Essential Orthopedics by Maheshwari & Mhaskar, 5th Edition, Page No: 134

77. Which of the following injury is the condylar fracture of tibia most commonly associated with?
a. Ligament injury
b. Meniscal injury
c. Peroneal nerve neuropraxia
d. Popliteal artery injury

Ans: b
Ref: Ebnezar Orthopedics 4th Edition, Page No: 264
Proximal tibia consists of the medial and lateral condyles along with the upper tibial articular surface and includes the
proximal 10-12 cm of the tibia.
Associated injuries with condylar fractures:
 Meniscal injury—50 percent
 Ligament injury—30 percent
 Peroneal nerve neuropraxia
 Popliteal artery injury

78. Which of the following is the most sensitive finding for cauda equina syndrome?
a. Decreased anal sphincter tone
b. Saddle anesthesia
c. Urinary retention
d. Weakness or numbness in the low extremities

Ans: c
Ref: Tintinalli’s Emergency Medicine: A ComprehensiveStudy Guide. 7th ed. New York, NY: McGraw-Hill; 2011;
Chapter 276.
Urinary retention with overflow incontinence is the most sensitive findingfor cauda equina syndrome (90%).

Paradise Institute & Self Study Centre Page 26


PARADISE WEEKLY MODEL TEST CEE MD/MS BASED MAGH 16

Cauda Equina Syndrome is caused by severe compression of the nerve roots in the thecal sac of the lumbar spine, most
commonly due to an acute lumbar disc herniation

79. Which part of the bone is Weinberger’s line present?


a. Diaphysis
b. Epiphysis
c. Metaphysis
d. Border of epiphysis and metaphysic

Ans: b
Ref: Ebnezar Orthopedics 4th Edition, Page No: 539
The radiological lines in scurvy
 White line of Frankel—dense line between epiphysis and metaphysis.
 Scurvy line—dense line within the metaphysis.
 Weinberger’s line—dense line within the epiphysis.
 Pelkan spurs—a bone spur from the lateral border of the metaphysis.

80. Which of the following is not a component of crush syndrome?


a. Myohemoglobinuria
b. Massive crushing of muscles
c. Acute tubular necrosis
d. Bleeding diathesis

Ans: d
Ref: Essential Orthopedics by Maheshwari & Mhaskar, 5th Edition, Page No: 43
Crush syndrome is seen when a limb is compressed for many hours resulting in massive crushing of muscles and release of
large amount of myohemoglobin which leads to acute tubular necrosis and renal failure.
Ophthalmology

81. Corneal transparency is maintained by:


a. Keratocyte
b. Bowman's membrane
c. Descemet’s membrane
d. Endothelium

Ans: d
Ref: AK Khurana Ophthalmology 6th Edition, Page No: 96, 97
Corneal transparency
The corneal transparency is the result of:
 Peculiar arrangement of corneal lamellae (lattice theory of Maurice).
 Peculiar refractive index of corneal lamellae with variation less than 200 mm (Goldmann and Benedek theory).
 Avascularity of cornea.
 Relative state of dehydration (78% water content), which is maintained by the barrier effects of epithelium and
endothelium and the active Na+ K+ ATPase pump of the endothelium.
 Swelling pressure (SP) of the stroma which counters the imbibition effect of intraocular pressure (IOP).
 Corneal crystallins, i.e, water soluble proteins of keratocytes (transketolase and aldehyde dehydrogenase class
IA1) also contribute to corneal transparency at the cellular level.

82. Which rectus muscle is the shortest?


a. Inferior rectus
b. Lateral rectus
c. Medial rectus
d. Superior rectus

Paradise Institute & Self Study Centre Page 27


PARADISE WEEKLY MODEL TEST CEE MD/MS BASED MAGH 16

Ans: a
Ref: Khurana Ophthalmology, 6th edition, page 336
All the four recti run forward around the eyeball and are inserted into the sclera, by flat tendons (about 10 mm broad) at
different distances from the limbus as under:
• Medial rectus: 5.5 mm
• Inferior rectus : 6.5 mm
• Lateral rectus : 6.9 mm
• Superior rectus : 7.7 mm

83. In which of the following, intraocular pressure is very high and inflammation is minimum?
a. Glaucomatocyclitic crisis
b. Acute iridocyclitis
c. Hypertensive uveitis
d. Angle closure glaucoma

Ans: a
Ref: AK Khurana Ophthalmology, Page No: 160
Glaucomatocyclitic Crisis (Posner-Schlossman syndrome) is a unilateral recurrent non-granulomatous iritis that is
associated with an elevated ocular pressure during the attacks. The self-limiting condition tends to occur in persons during
the 3rd to 6th decade and the visual fields, the optic nerve head, and anterior chamber angle are normal. A mild
inflammatory reaction is very rarely present as evidenced by a few keratic precipitates on the posterior surface of the
cornea. The cause of the glaucoma remains unknown, but a trabeculitis is suspected. Many patients (55%) subsequently
develop open angle glaucoma.

84. Photodynamic therapy is used for the treatment of:


a. Cataract
b. Glaucoma
c. Wet ARMD
d. Uveitis

Ans: c
Ref: AK Khurana Ophthalmology 6th Edition, Page No: 293, 295, 297
Age-related macular degeneration (ARMD), also called senile macular degeneration, is a bilateral disease of persons over
50 years of age. It is a leading cause of blindness in developed countries, in population above the age of 65 years. It is of
two types non-exudative and exudative. Photodynamic therapy (PDT) is the treatment of choice after anti-VEGF
injections for subfoveal and juxtafoveal classic CNVM. In PDT, vertiporfin, a photosensitizer or light activated dye is
injected intravenously.
Central serous chorioretinopathy (CSCR) is characterized by spontaneous serous detachment of neurosensory retina in the
macular region, with or without retinal pigment epithelium detachment. Photodynamic therapy (PDT) may be beneficial
for those with severe disease not amenable to conventional laser treatment, e.g., with sub-foveal leaks and chronic cases.

85. Giant papillary conjunctivitis is caused by:


a. Contact lens
b. HSV
c. Protruding corneal sutures
d. Tuberculosis

Ans: a
Ref: AK Khurana Ophthalmology 6th Edition, Page No: 82
Giant papillary conjunctivitis
 GPC is the inflammation of conjunctiva with formation of very large sized papillae.
 GPC, also known as mechanically-induced papillary conjunctivitis, is a localised allergic response to a physically
rough or deposited surface (contact lens, prosthesis, exposed nylon sutures and scleral buckle).
 Probably, it is a sensitivity reaction to components of the plastic leached out by the action of tears.

Paradise Institute & Self Study Centre Page 28


PARADISE WEEKLY MODEL TEST CEE MD/MS BASED MAGH 16

86. Polychromatic lustre is seen in:


a. Complicated cataract
b. Diabetic cataract
c. Post radiation cataract
d. Congenital cataract

Ans: a
Ref: AK Khurana Ophthalmology 6th Edition, Page No: 157
Complicated cataractmay develop as a complication of persistent iridocyclitis.
 Typical features of a complicated cataract in early stage are ‘polychromatic luster’ and ‘bread-crumb’ appearance
of the early posterior subcapsular opacities.
 In the presence of posterior synechiae, the complicated cataract progresses rapidly to maturity.

87. In electroretinogram, Negative ‘a’ wave represents the activity in:


a. Rods and cones
b. Inner retinal layers
c. Pigmentary epithelium
d. Lens channels

Ans: a
Ref: Parson Ophthalmology 22ndEdition, Page No: 614
• Negative ‘a’ wave represents the activity in rods and cones
• Positive ‘b’ wave arises in inner retinal layers
• Positive ‘c’ wave is associated with pigmentary epithelium
In electrooculogram (EOG) Arden index, i.e. ratio of light peak over dark through:
• 185–normal
• 150 to 185–borderline
• < 150–abnormal
Hemeralopia or day blindness is seen in cases of central nuclear or polar cataract, central corneal opacity, central vitreous
opacity and congenital deficiency of cones.
Dermatology

88. Major permeability barrier in skin is:


a. Epidermis
b. Dermis
c. Hypodermis
d. Muscular layer

Ans: a
Ref: Fitzpatrick Dermatology 8th Edition, Page No: 58
Three major layers—epidermis, dermis, hypodermis:
 Epidermis: major permeability barrier, innate immune function, adhesion, and ultraviolet protection.
 Dermis: major structural element, three types of components—cellular, fibrous matrix, and diffuse and filamentous
matrix. Also site of vascular, lymphatic, and nerve networks.
 Hypodermis (subcutis): insulation, mechanical integrity, containing the larger source vessels and nerves.

89. A 44-year-old woman presents with a 6-month history of nail changes. On examination all 20 nails are abnormal
with yellow discoloration and onycholysis. You suspect a diagnosis of yellow nail syndrome. Which of the following
features is not known to be associated with this disorder:
a. bronchiectasis
b. recurrent sinus infections
c. pleural effusions
d. flushing.

Paradise Institute & Self Study Centre Page 29


PARADISE WEEKLY MODEL TEST CEE MD/MS BASED MAGH 16

Ans: d
Ref: Fitzpatrick Dermatology 8th Edition
Yellow nail syndrome is a rare disorder that tends to present in middle-aged adults. Patients present with yellow-green
discolouration of thenails, most pronounced at the lateral margins. The nails also thicken,nail growth slows and
onycholysis may occur. It is associated withlymphoedema which may be of the extremities, face or genitals.30% of
patients develop pleural effusions, others develop recurrentbronchitis, chronic sinusitis or pneumonia. The disorder has
alsobeen associated with internal malignancy, immunodeficiency andrheumatoid arthritis. In yellow nail syndrome the
changes arepermanent and there is no effective treatment. Flushing is not afeature of the yellow nail syndrome.

90. Tacrolimus (FK-506) is:


a. Sulfonamide antibiotic
b. Tetracycline antibiotic
c. Macrolide antibiotic
d. Penicillin antibiotic

Ans: c
Ref: Fitzpatrick Dermatology 8th Edition, Page No: 226
 Tacrolimus (FK-506) is a macrolide antibiotic, derived from the bacteria Streptomyces tsukubaensis that, by
binding to immunophilin (FK506-binding protein), creates a complex that interacts and inhibits calcineurin, thus
blocking both T-lymphocyte signal transduction and IL-2 transcription.
 Pimecrolimus is also a calcineurin inhibitor and works in a manner similar to tacrolimus and CsA.

91. Pyoderma gangrenosum (PG) is caused by:


a. Sterile lymphocytic infiltration of the skin
b. Sterile macrophagic infiltration of the skin
c. Sterile neutrophilic infiltration of the skin
d. Sterile eosinophilic infiltration of the skin

Ans: c
Ref: Fitzpatrick Dermatology 8th Edition, Page No: 371
PYODERMA GANGRENOSUM AT A GLANCE
 Pyoderma gangrenosum (PG) is a rare inflammatory disease of unknown etiology characterized by sterile
neutrophilic infiltration of the skin. Similar neutrophilic infiltrations may occur in other organs. It is considered to
be one of the groups of neutrophilic dermatoses and clinical and histological overlap with some of these may
occur.
 PG is more frequent in female patients and occurs at any age, but usually between 40 and 60 years.
 The majority of patients with PG have other systemic diseases (such as arthritis, inflammatory bowel disease,
hematological dyscrasias, malignant disease, etc.), but PG occurs independently of these disorders.
 PG may present as ulcerative, bullous, pustular, or vegetative variants. Clinical features of different variants
sometimes overlap in individual patients but usually one variant dominates the clinical picture.
 There is no laboratory test or investigation that establishes the diagnosis of PG with certainty. The
histopathological findings are not diagnostic but can be supportive of the diagnosis of PG in the appropriate
clinical setting and are essential to rule out alternative diagnoses.
 The mainstays of management are systemic immunosuppressive agents together with appropriate local and topical
therapy.
 Ulcerative PG is a chronic disease. Remission usually requires months of treatment; maintenance therapy is
necessary in many and relapses are common. Significant morbidity and mortality are experienced by patients with
ulcerative and bullous PG.

92. A 34-year-old known alcoholic presents with severe abdominal pain radiating through to the back. Blood tests
reveal a significantly raised serum amylase. On examination he has large ecchymosis-likelesions bilaterally on his
flanks. What is the likely diagnosis:
a. pancreatic panniculitis
b. Cullen’s sign
Paradise Institute & Self Study Centre Page 30
PARADISE WEEKLY MODEL TEST CEE MD/MS BASED MAGH 16

c. Addison’s related hyperpigmentation


d. Grey-Turner’s sign.

Ans: d
Ref: Fitzpatrick Dermatology 8th Edition, Page No: 431
In this question the patient is presenting with Grey-Turner’s sign– bilateral bruising of the flanks. This is a cutaneous sign
of acutehaemorrhagic pancreatitis and carriers a grave prognosis. Other causesof Grey-Turner’s sign include blunt
abdominal trauma, rupturedaortic aneurysm and ruptured ectopic pregnancy.
Cutaneous features of pancreatitis:
• Grey-Turner’s sign
• Cullen’s sign – black-blue bruising around the umbilicus
• pancreatic panniculitis – tender, fluctuant nodules on the lower legs
• jaundice
• livedo reticularis
• urticaria
• thrombophlebitis migrans – pancreatic malignancy associated.

93. A child presents with recurrent episodes of lip edema and laryngeal edema and abdominal pain associated with
stress. Levels of which of the following are altered?
a. C1 esterase inhibitor
b. C5a
c. C3
d. C9

Ans: a
Ref: Fitzpatrick Dermatology 8th Edition, Page No: 847
Angioedema is never persistent, but rather shows complete resolution between acute episodes; and patients may show
increased levels of C1 esterase inhibitor. Cellulitis from infections of the anterior teeth may lead to lip swelling, but these
are acute and painful and the offending tooth is always identified.

Paradise Institute & Self Study Centre Page 31


PARADISE WEEKLY MODEL TEST CEE MD/MS BASED MAGH 16

94. What is the most specific test for syphilis?


a. VDRL
b. RPR
c. FTA-Abs
d. Kahn’s test

Ans: c
Ref: Fitzpatrick Dermatology 8th Edition, Page No: 2488, 2489
Treponemal serologic tests:
 Examples of treponemal serologic tests include the T. pallidum particle agglutination (TPPA) test, the
microhemagglutination assay for T. pallidum (MHA-TP), the fluorescent treponemal antibody absorption assay
(FTA-ABS), the T. pallidum haemagglutination test (TPHA), and various treponemal enzyme immunoassays
(EIAs) and immunochemiluminescence assays.
 These tests, which use whole or fragments of T. pallidum as antigen, directly detect infection with T. pallidum.
Compared with nontreponemal tests, they are more cumbersome to perform—except for treponemal EIAs—but
have greater sensitivity in the primary and late stages and slightly higher specificity.
Nontreponemal serologic tests
 The two most widely used nontreponemal tests are the Venereal Disease Research Laboratory (VDRL) and rapid
plasma reagin (RPR) tests.
 The VDRL and RPR begin to become reactive 4–5 weeks after infection, with 100% sensitivity by approximately
12 weeks, and revert to nonreactive in 25%–30% of cases during late latent syphilis.

Anesthesia

95. A 34-year-old woman is admitted to the ICU within 24 hours after sustaining burns over 80% of her body
surface area. Which of the following is most likely increased in this 24-hour period?
Paradise Institute & Self Study Centre Page 32
PARADISE WEEKLY MODEL TEST CEE MD/MS BASED MAGH 16

a. Metabolism
b. Vascular permeability
c. Oxygen tension
d. Urine output

Ans: b
Vascular permeability increases in the initial phase after a burn injury.

96. A 30-year-old male is undergoing a laparotomy for a ruptured spleen, liver laceration and bowel injury
following a road traffic accident. The estimated blood loss so far is 4L. During the procedure 8 units of packed red
cells and 3L of Hartmann’s solution have been infused. There is increased bleeding from the wound edge and from
the site of venous access. The following laboratory test supports the diagnosis of DIC rather than dilutional
coagulopathy:
a. Haemoglobin
b. D-Dimer
c. Platelet count
d. Bleeding time

Ans: b
Ref: Morgan & Mikhail’s Clinical Anesthesiology, 5th Edition, Page No: 704
Fibrinolysis is an important component of disseminated intravascular coagulation (DIC). Breakdown products of fibrin,
fibrin degradation products (FDPs) and D-Dimers are therefore elevated.
The specificity of these tests is, however, limited because other conditions such as venous thrombo-embolism, trauma and
recent surgery can lead to elevated FDPs and D-Dimers. The ongoing consumption of coagulation factors leads to elevated
global clotting times (aPTT and PT).
The diagnosis of DIC should be based on both clinical history and laboratory tests. The laboratory tests include FDPs, D-
Dimers, fibrinogen level, PT and aPTT. D-Dimers would not be elevated in dilutional coagulopathy.

97. A 67-year-old man is scheduled for surgical debridement of a leg burn suffered 5 days ago. In the preoperative
holding area, he receives midazolam and fentanyl. After induction with propofol and succinyl- choline, the patient’s
continuous electrocardiogram shows peaked T waves. Which of the following agents most likely caused the electro-
cardiogram changes?
a. Midazolam
b. Fentanyl
c. Propofol
d. Succinylcholine

Ans: d
Ref: Morgan & Mikhail’s Clinical Anesthesiology, 5th Edition, Page No: 733
The peaked T waves on ECG are suggestive of an elevated potassium level. Succinylcholine can produce an exaggerated
hyperkalemic response in burn-injured patients.

98. Vecuronium is:


a. Very short acting
b. Short acting
c. Intermediate acting
d. Long acting

Ans: c
Ref: Ajay Yadav Anesthesia 6th Edition, Page No: 128
Non-depolarizing muscle relaxants

Paradise Institute & Self Study Centre Page 33


PARADISE WEEKLY MODEL TEST CEE MD/MS BASED MAGH 16

99. Which of the following is centrally acting muscle relaxant?


a. Chlorpropamide
b. Indomethacin
c. Botulinum
d. Tizanidine

Ans: d
Ref: Ajay Yadav Anesthesia 6th Edition, Page No: 133
The commonly used centrally acting muscle relaxants are:
 Chlorzoxazone, chlormezanone
 Diazepam
 Baclofen
 Tizanidine, metaxalone
Others like mephenesin, meprobamate are no more used intravenously (mephenesin is still used in ointments).

100. Inhalational agent of choice for hepatic patients is:


a. Halothane
b. Nitrous oxide
c. Sevoflurane
d. Propofol

Ans: c
Ref: Ajay Yadav Anesthesia 6th Edition, Page No: 201
Sevoflurane is the inhalational agent of choice for hepatic patients while halothane and nitrous oxide should be avoided.

101. Which of the following anesthetics techniques is associated with an increased incidence of intraoperative
awareness?
a. Sevoflurane anesthesia
b. Propofol infusion
c. Fentanyl infusion
d. Desflurane anesthesia

Ans: c
Fentanyl infusion
Primarily narcotic-based techniques have been shown to be associated with a higher incidence of intraoperative awareness
than inhalation anesthetic techniques. This is because narcotic-based techniques do not provide for amnesia. Anytime a
narcotic is used for the primary anesthetic agents, supplemental amnesic agents should be administered as well.

Psychiatry

Paradise Institute & Self Study Centre Page 34


PARADISE WEEKLY MODEL TEST CEE MD/MS BASED MAGH 16

102. Separation anxiety disorder is usually diagnosed between:


a. 3 months to 6 months
b. 2 years to 3 years
c. 3 years to 5 years
d. 7 years to 9 years

Ans: d
Ref: USMLE Step 1 First Aid 2020, Page No: 557
Separation anxiety disorder
 Overwhelming fear of separation from home or attachment figure lasting ≥ 4 weeks
 Can be normal behavior up to age 3–4
 May lead to factitious physical complaints to avoid school
 Treatment: CBT, play therapy, family therapy
 Most common onset is between 7-9 years of age

103. Treatment of postpartum blues is:


a. Supportive care
b. CBT
c. SSRIs
d. Antipsychotics

Ans: a
Ref: USMLE Step 1 First Aid 2020, Page No: 562
Peripartum mood disturbances
Onset during or shortly after pregnancy or within 4 weeks of delivery. Increased risk with history of mood disorders.
Maternal (postpartum) blues
 50–85% incidence rate
 Characterized by depressed affect, tearfulness, and fatigue starting 2–3 days after delivery
 Usually resolves within 2 weeks
 Treatment: supportive
 Follow up to assess for possible MDD with peripartum onset
MDD with peripartum onset
 10–15% incidence rate. Formerly called postpartum depression
 Meets MDD criteria with onset no later than 1 year after delivery
 Treatment: CBT and SSRIs are first line
Postpartum psychosis
 0.1–0.2% incidence rate
 Characterized by mood-congruent delusions, hallucinations, and thoughts of harming the baby or self
 Risk factors include first pregnancy, family history, bipolar disorder, psychotic disorder, recent medication change
 Treatment: hospitalization and initiation of atypical antipsychotic; if insufficient, ECT may be used

104. Precontemplation means:


a. Denying problem
b. Acknowledging problem, but unwilling to change
c. Preparing for behavioral changes
d. Maintaining changes

Ans: a
Ref: USMLE Step 1 First Aid 2020, Page No: 568
Stages of change in overcoming addiction
 Precontemplation—denying problem
 Contemplation—acknowledging problem, but unwilling to change
 Preparation/determination—preparing for behavioral changes

Paradise Institute & Self Study Centre Page 35


PARADISE WEEKLY MODEL TEST CEE MD/MS BASED MAGH 16

 Action/willpower—changing behaviors
 Maintenance—maintaining changes
 Relapse—(if applicable) returning to old behaviors and abandoning change

105. Treatment of neuroleptic malignant syndrome is:


a. Phentolamine
b. Lithium
c. Dantrolene
d. Clozapine

Ans: c
Ref: USMLE Step 1 First Aid 2020, Page No: 569
Neuroleptic malignant syndrome
 Cause: Antipsychotics (typical > atypical) + genetic predisposition
 Manifestation:Malignant FEVER: Myoglobinuria, Fever, Encephalopathy, Vitalsunstable, • Enzymes (eg,
 CK), muscle Rigidity (“lead pipe”)
 Treatment: Dantrolene, dopamine agonist (eg, bromocriptine), discontinue causative agent

106. The benzodiazepines’ action depends on their interaction with which of the following receptors?
a. GABA
b. Serotonin
c. N-methyl-D-aspartate- (NMDA)-glutamate
d. Acetylcholine

Ans: a
Ref: (Kaplan and Sadock, pp 948-950.)
Benzodiazepines bind to GABA receptors, which represent the main cortical and thalamic inhibitory system and potentiate
the response of these receptors to GABA. Benzodiazepines do not have any direct effect on the GABA receptors unless
GABA is present.

107. Venlafaxine is:


a. Selective serotonin reuptake inhibitor
b. Serotonin norepinephrine reuptake inhibitor
c. Tricyclic antidepressant
d. Monoamine oxidase inhibitor

Ans: b
Ref: USMLE Step 1 First Aid 2020, Page No: 575
 Selective serotonin reuptake inhibitors: Fluoxetine, fluvoxamine, paroxetine, sertraline, escitalopram, citalopram.
 Serotonin norepinephrine reuptake inhibitors: Venlafaxine, desvenlafaxine, duloxetine, levomilnacipran,
milnacipran.
 Tricyclicantidepressants:Amitriptyline, nortriptyline, imipramine, desipramine, clomipramine, doxepin,
amoxapine.
 Monoamine oxidase inhibitors: Tranylcypromine, Phenelzine, Isocarboxazid, Selegiline

108. A 22-year-old woman is seen by a psychiatrist in the ED after she is found walking in the middle of a busy
street with no shoes on. During her interview she is asked to tell the physician what the following statement means:
“Those in glass houses should not throw stones.” Which of the following best describes the cognitive functions being
tested by this request?
a. Orientation
b. Immediate memory
c. Fund of knowledge
d. Abstract reasoning

Paradise Institute & Self Study Centre Page 36


PARADISE WEEKLY MODEL TEST CEE MD/MS BASED MAGH 16

Ans: d
Ref: Kaplan and Sadock, p 203
Abstract reasoning is the ability to think flexibly, creatively, and logically. It is typically tested during a mental status
examination by asking a patient to interpret a proverb. Having a patient subtract 7s from 100 tests concentration.
Orientation is evaluated by asking the patient whether he knows where he is, who he is, and what the date is. Immediate
memory is tested by asking a patient to repeat a series of numbers immediately after you say them with no time delay.
Radiology

109. Which of the following is true about color Doppler study?


a. Red means flow towards probe and blue means flow away from the probe.
b. Red means arterial flow and blue means venous flow
c. Blue means flow towards probe and red means flow away from the probe.
d. Blue means arterial flow and red means venous flow

Ans: a
Requisites Ultrasound, Page No 16
Color Doppler ultrasonography is sensitive to Doppler signals throughout an adjustable portion of the field of view.
It provides a real-time image displaying tissue morphology in grayscale and blood flow in color.
Color Doppler ultrasonography analyzes the phase information, frequency, and amplitude of the returning echoes.
Signals from moving red blood cells are assigned a color (red vs. blue) based on the direction of the phase shift (i.e., the
direction of blood flow toward = red or away from the transducer = blue).

110. In the ultrasound deeper tissues produce weaker signal and they are electronically amplified after they return
to the transducer. What is this called?
a. Doppler setting
b. Filter setting
c. Time gain compensation (TGC) setting
d. Frame per second (fps) setting

Ans: c
RequisitesUltrasound, Page No: 11
 Because of sound attenuation, an interface in the deep tissues will produce a weaker reflection than a similar
interface in the near tissues.
 To compensate for this, signals from deeper tissues are electronically amplified after they return to the transducer.
 Because the depth of the interface is determined by the amount of time it takes for the transmitted sound pulse to
return to the transducer, this variable amplification is referred to as the time gain compensation (TGC).
 The amount of gain is shown to the side of the image in the form of a line or curve.
 The TGC curve is moved a variable amount to the right (indicating increased gain) in the deeper aspects of the
image.
 Because different tissues attenuate sound to a different degree, the TGC curve requires frequent readjustments as
different structures are scanned.

111. Which of the following mode is particularly important in studying cardiac valve and wall motion and in
documenting fetal heart rate and activity?
a. D mode
b. A mode
c. B mode
d. M mode

Ans: d
RequisitesUltrasound, Page No: 8
The M-mode is designed to document and analyze tissue motion. Using the 2D image as a guide, a particular scan line is
selected to correspond to the moving structure of interest.

Paradise Institute & Self Study Centre Page 37


PARADISE WEEKLY MODEL TEST CEE MD/MS BASED MAGH 16

The reflections from this scan line are then displayed in agraphic form, with motion on the vertical axis and time on the
horizontal axis. This mode is particularly important in studying cardiac valve and wall motion and in documenting fetal
heart rate and activity.

112. Which of the following is the most commonly used method to evaluate vesico-ureteric reflux ( VUR)?
a. Triple phase CT
b. CT IVU
c. IVU
d. MCUG

Ans: d
The requisites, Pediatric Imaging, Page No: 119
Micturating cystourethrography (MCUG) or Voiding cystourethrography (VCUG) is the most commonly used as well as
the optimal method to evaluate for vesicoureteral reflux (VUR), assess bladder anatomy and function, and judge urethral
anatomy.

113. A 4 months old child with a long history of abdominal distension and constipation has an X-ray showing low
bowel loop obstruction. Barium enema shows narrowed caliber of the rectum with abrupt transition. Rectosigmoid
ratio is < 0.9. What is the most likely diagnosis?
a. Hirschsprung Disease
b. High anorectal malformation
c. Meconium plug syndrome
d. Meconium ileus

Ans a
The Requisites, Pediatric Imaging, Page No: 92
Hirschsprung disease, or colonic aganglionosis is characterized by an absence of ganglion cells inthe myenteric plexus,
probably caused by failure of craniocaudal migration of neuroblasts between the 7th and 12th weeks of gestation. In 85%
of cases, the aganglionosis involves the distal sigmoid and rectum, whereas in the remainder, the colon lacks ganglion cells
proximal to the splenic flexure.
The rectosigmoid index can also be used to evaluate for a transition zone. In the normal patient, the rectum is usually wider
than the sigmoid colon; however, the opposite is true in patients with Hirschsprung disease. A rectosigmoid ratio greater
than 0.9 excludes the diagnosis of rectosigmoid Hirschsprung disease.

114. Box-shaped heart is seen in:


a. Ebstein anomaly
b. Tricuspid atresia
c. TAPVR
d. TOF

Ans: a
Ref: Box-shaped heart by Daniel J Bell in Rdaiopedia.
A box-shaped heart is a radiographic description given to the cardiac silhouette in some cases of Ebstein anomaly. The
classic appearance of this finding is caused by the combination of the following features:
 huge right atrium that may fill the entire right hemithorax
 shelved appearance of the left cardiac contour due to the dilated right ventricular outflow tract or elevated residual
right ventricle
 small aorta and main pulmonary artery

115. Ribbons ribs are described in:


a. NF Type 1
b. NF Type 2
c. Sturge Weber syndrome
d. MEN Type 1

Paradise Institute & Self Study Centre Page 38


PARADISE WEEKLY MODEL TEST CEE MD/MS BASED MAGH 16

Ans: a
Ref: Ribbon ribs deformity YurangaWeerakkody in Radiopedia.
Ribbon ribs deformity refers to the presence of thinned ribs on image studies. Such findings could be present in patients
with neurofibromatosis type 1 due to the presence of multiple neurofibromas of intercostal nerves.
This deformity is also described in:
 Osteogenesis imperfecta
 Edwards syndrome (trisomy 18)
 Gorham disease

Anatomy

116. Which of the following is not a saddle joint?


a. Carpometacarpal joint of thumb
b. Metacarpophalangeal joint
c. Ankle joint
d. Calcaneocuboid joint

Ans: b
Ref: Gray’s anatomy, 41st Edition; Page No: 101
 Saddle joints are biaxial joints in which the articular surfaces have both concave and convex regions. Each surface
is maximally convex in one direction and maximally concave in another, at right angles to the first. The convexity
of the larger surface is apposed to the concavity of the smaller surface and vice versa. The most familiar saddle
joint is the carpometacarpal joint of the thumb; other examples include the ankle and calcaneocuboid joints.
Plane joints
 Plane joints, such as intermetatarsal and some intercarpal joints, have almost flat surfaces. Slight curvature is often
disregarded, although it is usual, and movements are considered to be pure translations or sliding between bones.
Hinge joints
 These resemble hinges because movement takes place about a single stationary axis, and so is largely restricted to
one plane. Examples are interphalangeal and humero-ulnar joints. The condyles may lie within a common fibrous
capsule (as in the knee), or in separate capsules that necessarily cooperate in all movements as a condylar pair (as
in the temporomandibular joints).
Pivot joints
 These are uniaxial joints in which an osseous pivot inside an osteoligamentous ring allows rotation only around the
axis of the pivot. Pivots may rotate in rings (e.g. the head of the radius rotates within the anular ligament and ulnar
radial notch), or rings may rotate around pivots (e.g. the atlas rotates around the dens of the axis).
Ellipsoid joints
 Ellipsoid joints are biaxial, and consist of an oval, convex surface apposed to an elliptical concavity. Examples are
the radiocarpal and metacarpophalangeal joints. A primary movement occurs around two orthogonal axes, such as
flexion–extension and abduction–adduction, and may be combined as circumduction. Rotation around the third
axis is largely prevented by general articular shape.
Ball-and-socket joints
 These multiaxial joints are formed by a globoid ‘head’ articulating with an opposing cup. Prime examples are the
hip and shoulder joints. Although their surfaces resemble parts of spheres, they are not strictly spherical but
slightly ovoid, and consequently congruence is not perfect in most positions. Indeed, it occurs in only one position,
at the end of the most common movement.

117. A5-year-old boy sustains a tear in his gastrocnemius muscle when he is involved in a bicycle accident.
Regeneration of the muscle will occur through which one of the following mechanisms?
a. Differentiation of satellite cells
b. Dedifferentiation of myocytes into myoblasts
c. Fusion of damaged myofibers to form new myotubes
d. Hyperplasia of existing myofibers

Paradise Institute & Self Study Centre Page 39


PARADISE WEEKLY MODEL TEST CEE MD/MS BASED MAGH 16

Ans: a
Ref: (Kierszenbaum, p 209-210. Kumar, p 82, 85-86. Alberts, p 1466.)
Satellite cells in skeletal muscle proliferate and reconstitutethe damaged part of the myofibers. They are supportive cells
for maintenance of muscle and a source of new myofibers after injury or after increased load. There is no dedifferentiation
of myocytes into myoblasts, or fusion of damaged myofibers to form new myotubes. Hypertrophy, not hyperplasia, occurs
in existing myofibers in response to increased load. Proliferation of fibroblasts may occur in the damaged area but leads to
fibrosis, not repair of skeletal muscle. Fibroblasts do not differentiate into myocytes. The multinucleate organization of
skeletal muscle is derived developmentally by fusion and not by amitosis (failure of cytokinesis after DNA synthesis).
Mitotic activity is terminated after fusion occurs. In the development of skeletal muscle, myoblasts of mesodermal origin
undergo cell proliferation. Myocyte cell division ceases soon after birth. Myoblasts, which are mononucleate cells, fuse
with each other end to end to form myotubes.
This process requires cell recognition between myoblasts, alignment, and subsequent fusion.

118. A neurologist uses a pin to test the sensation to a 26-year-old man’s scalp just near the hair line anteriorly.
Which of the following nerves provides the innervation to the scalp in this region?
a. CN V
b. CNVII
c. CNX
d. Spinal nerve C2

Ans: a
Ref: Grays Anatomy 41st edition page 289
The anterior scalp is innervated by CN V, whereas the posterior scalp is innervated by spinal nerve C2.
The scalp is the unit of tissue that covers the calvaria. The tissue is composed of five layers and can be
remembered by the acronym SCALP. Most superficial is the skin, which includes the dermis and the superficial fascia.
Deep to that is a layer of dense connective tissue that binds tightly to the skin.The next layer is the aponeurosis of the
occipitofrontalis muscle (galea aponeurotica). These three layers adhere tightly and move together as a unit. The fourth
layer is one of loose connective tissue. The fifth layer is the periosteum, which covers the bone itself. The periosteum
adheres tightly to the bone, especially in the region of the cranial sutures. The flexibility of the loose connective tissue
allows the more superficial layers to move over the periosteum. In infants, the periosteum is adherent to the sutures.
The nerves of the scalp anteriorly are from the first and third divisions of the trigeminal nerve. Medially, the
supraorbital and supratrochlear nerves supply sensory innervation. Laterally, the auriculotemporal nerve pro- vides sensory
innervation. The posterior scalp is supplied medially by the pos- terior primary rami of cervical spinal nerves (C2, as the
greater occipital nerve, and C3). Laterally, the skin is supplied by anterior primary rami that form the cervical plexus, in
particular the lesser occipital and posterior auricular nerves.

119. If a mass extends outside from the central canal of spinal cord, which of the following part is involved the
earliest?
a. Arm
b. Trunk
c. Thigh
d. Leg

Ans: a
Ref: Grays Anatomy 41st edition page 300

Paradise Institute & Self Study Centre Page 40


PARADISE WEEKLY MODEL TEST CEE MD/MS BASED MAGH 16

120. A35-year-old man developed an intracranial hemorrhage when one of the meningeal arteries ruptured.
Anatomically, where is the hematoma located?
a. Immediately superficial to the dura
b. Immediately deep to the dura
c. Within the subarachnoid space
d. Within the brain parenchyma

Ans: a
Ref: Moore KL, Dalley AF. Clinically Oriented Anatomy, 5th ed. Baltimore, MD: Lippincott Williams & Wilkins,
2006:908–21.
Injuries to the meningeal arteries lead to epidural hematomas.

121. Which of the following back muscles is innervated by dorsal rami?


a. Latissimus dorsi
b. Levator scapulae
c. Rhomboid major
d. Longissimus

Ans: d
Ref: Moore's Clinically Oriented Anatomy; 7th Edition, Page No: 484
The longissimus, a deep or intrinsic back muscle, is innervated by dorsal rami. All superficial or extrinsic back muscles
except the trapezius, which is innervated by the accessory nerve, are innervated by ventral rami.

122. The median nerve does which of the following?


a. Innervates the elbow joint with articular branches
b. Innervates the medial half of the flexor digitorum profundus
c. Innervates the hypothenar muscles
d. Innervates lumbricals 3 and 4

Ans: a
Ref: Moore's Clinically Oriented Anatomy; 7th Edition, Page No: 761

Paradise Institute & Self Study Centre Page 41


PARADISE WEEKLY MODEL TEST CEE MD/MS BASED MAGH 16

The median nerve assists in the innervation of the elbow joint and gives muscular branches to pronator teres, flexor carpi
radialis, palmaris longus, and flexor digitorum superficialis. The median nerve also has an anterior interosseous branch that
innervates the lateral part of the flexor digitorum profundus, flexor pollicis longus, and pronator quadratus.
The recurrent branch of the median nerve innervates the thenar muscles, and the palmar cutaneous branch innervates the
skin of the lateral part of the palm.

123. During transesophageal echocardiography (TEE), an ultrasound transducer is placed through the nose or
mouth to lie directly behind the heart. The closer a structure is to the transducer, the better the ultrasound image
that can be obtained. In TEE, which heart valve can be best visualized?
a. Tricuspid
b. Pulmonary
c. Mitral
d. Aortic

Ans: c
Ref: Gray’s Anatomy for Students, 181-208
The mitral valve is best visualized by TEE because the transducer within the esophagus is directly posterior to the left
atrium. The physical laws that apply to ultrasound imaging dictate that the closer the structure to the transducer, the better
the ability to obtain a better image. This question asks which heart valve is most directly related to the posterior aspect of
the left atrium, which is the mitral valve.

124. A 44-year-old man is admitted to the emergency department with excessive vomiting and dehydration.
Radiographic images demonstrate that part of the bowel is being compressed between the abdominal aorta and the
superior mesenteric artery. Which of the following intestinal structures is most likely being compressed?
a. Second part of duodenum
b. Transverse colon
c. Third part of duodenum
d. First part of duodenum

Ans: c
Ref: Gray’s Anatomy for Students, 331
• The third part of the duodenum takes a pathology situated anterior to the abdominal aorta and inferior to the
superior mesenteric artery (a major ventral branch of the abdominal aorta). Because the third part of the duodenum
lies in the angle between (“sandwiched”) these two structures, constrictions of this portion of the duodenum can
occur readily.
• The second part of the duodenum lies parallel with, and to the right of, the abdominal aorta and is not normally in
close proximity to the superior mesenteric artery.
• The transverse colon takes a horizontal pathology through the anterior abdominal cavity but travels superior or
anterior to the superior mesenteric artery.
• The first part of the duodenum continues from the pylorus, flexing to lead to the second part of the duodenum;
thus, it is not located near the superior mesenteric artery or abdominal aorta.

125. A 64-year-old man is diagnosed with an acute stroke. His main deficit is a partial loss of his visual field. The
neurologist diagnoses a lesion of the occipital lobe. Which of the following arteries is likely to be involved?
a. Internal carotid
b. External carotid
c. Middle cerebral
d. Posterior cerebral

Ans: d
Ref: Gray’s Anatomy for Students, 477
The occipital lobes are supplied by the posterior cerebral arteries, which are terminal branches of the basilar artery.

126. Calcaneal Tendon Reflex tests:

Paradise Institute & Self Study Centre Page 42


PARADISE WEEKLY MODEL TEST CEE MD/MS BASED MAGH 16

a. L3, L4
b. L4, L5
c. L5, S1
d. S1, S2

Ans: d
Ref: Moore's Clinically Oriented Anatomy; 7th Edition, Page No: 607
Calcaneal Tendon Reflex
• The ankle jerk reflex, or triceps surae reflex, is a calcaneal tendon reflex.
• It is a myotatic reflex elicited while the person’s legs are dangling over the side of the examining table. The
calcaneal tendon is struck briskly with a reflex hammer just proximal to the calcaneus.
• The normal result is plantarflexion of the ankle joint. The calcaneal tendon reflex tests the S1 and S2 nerve roots. If
the S1 nerve root is injured or compressed, the ankle reflex is virtually absent.

127. Strongest flexor of hip joint is:


a. Iliopsoas
b. Adductor magnus
c. Pectineus
d. Gluteus maximus

Ans: a
Ref: Moore's Clinically Oriented Anatomy; 7th Edition, Page No: 632
The main muscles producing movements of the hip joint are:
 The iliopsoas is the strongest flexor of the hip.
 In addition to its function as an adductor, the adductor magnus also serves as a flexor (anterior or aponeurotic part)
and an extensor (posterior or hamstrings part).
 Several muscles participate in both flexion and adduction (pectineus and gracilis as well all three “adductor”
muscles).
 In addition to serving as abductors, the anterior portions of the gluteus medius and minimus are also medial
rotators.
 The gluteus maximus serves as the primary extensor from the flexed to the straight (standing) position, and from
this point posteriorly, extension is achieved primarily by the hamstrings. The gluteus maximus is also a lateral
rotator.

Physiology

128. Sodium channels are specifically blocked by:


a. Nifedipine
b. Tetrodotoxin
c. Tetraethylammonium
d. Tetraethyl lead

Ans: b
Ref: Guyton and Hall Textbook of Medical Physiology, Elsevier; 13th Edition, Page No: 66
• Sodium channels can be blocked by a toxin called tetrodotoxin when it is applied to the outside of the cell
membrane where the sodium activation gates are located. Conversely, tetraethylammonium ion blocks the
potassium channels when it is applied to the interior of the nerve fiber.

129. Otoliths have which of the following properties?


a. They contain kinocilia and cation channels that are opened when pulled by extracellular filaments.
b. They are located only in the saccule and utricle.
c. They are located only in the semicircular canals.
d. They bend stereocilia toward but not away from the kinocilium.

Paradise Institute & Self Study Centre Page 43


PARADISE WEEKLY MODEL TEST CEE MD/MS BASED MAGH 16

Ans: b
Ref: Guyton and Hall Textbook of Medical Physiology, Elsevier; 13th Edition, Page No: 892
Otoliths are located in the otolithic organs, the saccule and utricle, not in the three semicircular canals. They are pulled by
gravity against the gelatinous cap containing stereocilia, which can bend either toward or away from the kinocilium. This
bending preferen- tially transduces linear acceleration rather than angular acceleration (which is transduced by hair cells in
the semicircular canals).

130. Pleural pressure positive in:


a. End of inspiration
b. End of expiration
c. End of forced expiration
d. Start or beginning of inspiration

Ans: c
Ref: Ganong Review of Medical Physiology, Lange; 25th Edition, Page No: 628
• Inspiration is an active process. The contraction of the inspiratory muscles increases intrathoracic volume. The
intrapleural 2.5 mm Hg (relative to atmospheric) at the start of inspiration,
• Inspiration is an active process. The contraction of the inspiratory muscles increases intrathoracic volume. The
intrapleural pressure at the base of the lungs, which is normally about –2.5 mm Hg (relative to atmospheric) at the
start of inspiration, decreases to about –6 mm Hg.
• Pleural pressure is always negative during normal quiet breathing. It becomes positive in case of forceful
breathing.

131. Inner hair cells depolarize due to:


a. Potassium efflux
b. Potassium influx
c. Sodium influx
d. Sodium efflux

Ans: b
Reference: Berne and Levy physiology; 7th edition; Page No 146-147
Sound Transduction
Paradise Institute & Self Study Centre Page 44
PARADISE WEEKLY MODEL TEST CEE MD/MS BASED MAGH 16

• In view of the wide range of frequencies and amplitudes of sound stimuli, it is no surprise that hair cell
transduction must provide for a fast response.
• The fast response to deflection of the cilia is based on direct opening of ion channels by so-called tip links that
connect the tip of each stereocilium with the shaft of the next taller one With deflection, the tip links are subjected
to a lever action that transiently opens the channels, allows the entry of K+ (because of the high [K+] and high
potential in endolymph), and depolarizes the hair cell.
• Several mechanisms have been proposed to account for the equally important rapid adaptation necessary for a
high-frequency response.
• A “spring” response by the tip links would allow the attachment point of the tip link to be moved along the
stereocilium’s shaft to reset the mechanical leverage of the tip link. In addition, it has been observed that Ca++ can
enter and bind to the open channel, change it to require greater opening force, and thereby reduce the statistical
probability of opening.
• The potential gradient that induces movement of ions into hair cells includes both the resting potential of the hair
cells and the positive potential of the endolymph. As noted previously, the total gradient across the apical
membrane of hair cells is about 140 mV. Therefore, a change in K+ conductance in the apical membranes of hair
cells results in a rapid current flow that produces the receptor potential in these cells.

132. Pancreatic islet B cells have:


a. GLUT 1
b. GLUT 2
c. GLUT 3
d. GLUT 4

Ans: b
Ref: Ganong Review of Medical Physiology, Lange; 25th Edition, Page No: 433

133. What is the total conduction delay in A-V node &bundle system of heart?
a. 0.04 second
b. 0.09 second
c. 0.13 second
d. 0.16 second

Ans: c
Ref: Guyton and Hall Textbook of Medical Physiology, Elsevier; 13th Edition, Page No: 125

Paradise Institute & Self Study Centre Page 45


PARADISE WEEKLY MODEL TEST CEE MD/MS BASED MAGH 16

 The impulse generated in SA node after traveling through the intermodal pathways, reaches the A-V node in about
0.03 second.
 Then there is a delay of another 0.09 second in the A-V node itself before the impulse enters the penetrating
portion of the A-V bundle, where it passes into the ventricles.
 A final delay of another 0.04 second occurs mainly in this penetrating A-V bundle, which is composed of multiple
small fascicles passing through the fibrous tissue separating the atria from the ventricles.
 Thus, the total delay in the A-V nodal and A-V bundle system is about 0.13 second.
 This delay, in addition to the initial conduction delay of 0.03 second from the sinus node to the A-V node, makes a
total delay of 0.16 second before the excitatory signal finally reaches the contracting muscles of the ventricle.

134. A motor unit that innervates only three muscle fibers is likely to innervate muscle in which of the following?
a. Back
b. Biceps
c. Bladder
d. Thumb

Ans: d
Ref: Ganong Review of Medical Physiology, Lange; 24th Edition
Motor units are smallest in the parts of the body that have the most precise motor control, such as the thumb, fingers, and
tongue.

135. Pacemaker regulating the rate of respiration is:


a. Pneumotaxic center
b. Dorsal group of nucleus
c. Apneustic center
d. Pre-Bötzinger

Ans: d
Ref: Ganong Review of Medical Physiology, Lange; 24th Edition, Page No: 658
Pacemaker of respiration is pre-Bötzinger complex of neurons, which is a part of VRG in medulla
 The main components of the respiratory control pattern generator responsible for automatic respiration are located
in the medulla.
 Rhythmic respiration is initiated by a small group of synaptically coupled pacemaker cells in the pre-
Bötzinger complex (pre-BÖTC) on either side of the medulla between the nucleus ambiguus and the lateral
reticular nucleus.
 These neurons discharge rhythmically, and they produce rhythmic discharges in phrenic motor neurons that are
abolished by sections between the pre-Bötzinger complex and these motor neurons. They also contact the
hypoglossal nuclei, and the tongue is involved in the regulation of airway resistance.

136. Which of the following antagonist is not used to treat chemotherapy-induced vomiting?
a. 5-HT3
b. D2
c. NK-1
d. H1

Ans: d
Ref: USMLE Step 1 First Aid 2020, Page No: 496
Vomiting center
 Coordinated by nucleus tractus solitarius (NTS) in the medulla, which receives information from the
chemoreceptor trigger zone (CTZ, located within area postrema in 4th ventricle), GI tract (via vagus nerve),
vestibular system, and CNS.
 CTZ and adjacent vomiting center nuclei receive input from 5 major receptors: muscarinic (M1), dopamine (D2),
histamine (H1), serotonin (5-HT3), and neurokinin (NK-1) receptors.
 5-HT3, D2, and NK-1 antagonists used to treat chemotherapy-induced vomiting.
Paradise Institute & Self Study Centre Page 46
PARADISE WEEKLY MODEL TEST CEE MD/MS BASED MAGH 16

 H1 and M1 antagonists treat motion sickness; H1 antagonists treat hyperemesis gravidarum.

137. A 28-year-old woman has a fever of 40°C as a result of influenza. Which of the following is likely to occur
during the fever?
a. Cutaneous vasoconstriction
b. Reduction of hypothalamic set-point temperature
c. Decrease in shivering
d. Increase in sweating

Ans: a
Ref: Guyton and Hall Textbook of Medical Physiology, Elsevier; 13th Edition, Page No: 374
Fever elevates the hypothalamic set-point temperature, activating heat conservation responses, which include cutaneous
vasoconstric- tion. Sweating is inhibited, and shivering occurs. There is a strong subjective sensation of cold, leading to
behavioral efforts to warm the body such as pulling on blankets.

138. Striatum means:


a. Putamen + Caudate
b. Putamen + Globus pallidus
c. Caudate + Globus pallidus
d. Putamen + Globus pallidus + Caudate

Ans: a
Ref: USMLE Step 1 First Aid 2020, Page No: 500
Striatum = putamen (motor) + Caudate (cognitive).
Lentiform = putamen + globus pallidus.

139. Mean arterial pressure (MAP) is an average blood pressure in an individual during a single cardiac cycle. This
is calculated as:
a. (SBP + 2DBP)/3
b. (DBP + 2SBP)/3
c. (SBP + 3DBP)/2
d. (DBP + 3SBP)/2

Ans: a
Ref: Ganong Review of Medical Physiology, Lange; 24th Edition, Page No: 544
Mean BP
= DBP + 1/3 pulse pressure
= DBP + 1/3 (SBP – DBP)
= (3DBP + SBP – DBP) /3
= 2/3 DBP +1/3 SBP = (SBP + 2DBP)/3

Pathology

140. A familial deficiency of glucose-6-phosphatase is most likely to be associated with the abnormal accumulation
of glycogen in which of the following locations?
a. Basal ganglia
b. Cardiac muscle
c. Liver cells
d. Skeletal muscle

Ans: c
Ref: Robbins and Cotran Pathologic basis of disease9/e
The glycogen storage diseases result from hereditary deficiencies involving any of the enzymes involved in the synthesis or
degradation of glycogen. Type I glycogen storage disease (von Gierke disease) results from a deficiency of glucose-6-
Paradise Institute & Self Study Centre Page 47
PARADISE WEEKLY MODEL TEST CEE MD/MS BASED MAGH 16

phosphatase, an enzyme that is necessary for the conversion of glucose 6-phosphate to glucose in the liver. This deficiency
results in the accumulation of glycogen in the cytoplasm of hepatocytes. Symptoms of this disorder result from recurrent
severe fasting hypoglycemia and include dizziness, sweating, and convulsions.

141. A man sustains a left femoral fracture in a skiing accident, and his leg is placed in a plaster cast. After the leg
has been immobilized for several weeks, the diameter of the left calf has decreased. This change is most likely to
result from which of the following alterations in the calf muscles?
a. Aplasia
b. Hypoplasia
c. Atrophy
d. Dystrophy

Ans: c
Ref: Robbins and Cotran Pathologic basis of disease9/e p36
 Reduced workload causes shrinkage of cell size because of loss of cell substance, a process called atrophy.
 Aplasia refers to lack of embryonic development; hypoplasia describes poor or subnormal development.
 Dystrophy of muscles refers to inherited disorders of skeletal muscles that lead to muscle weakness and wasting.

142. Which of the following enzymes is necessary for the final biochemical reaction involved in the activation of
vitamin D?
a. 1-α-Hydroxylase in the kidney
b. 7-α-Hydroxylase in the liver
c. 17-Hydroxylase in the adrenal cortex
d. 11-Hydroxylase in the adrenal cortex

Ans: a
Ref: Robbins and Cotran Pathologic Basis of Disease, 7th ed. Philadelphia, PA: Elsevier Saunders, 2004.
The main source of vitamin D is the conversion of 7-dehydrocholesterol to cholecalciferol by sunlight in the skin. This
reaction forms previt- amin D3 (cholecalciferol), which in the liver is converted to 25- hydroxy-D3 (calcifediol) by a
cytochrome P-450 mixed-function oxidase. Calcifediol is converted in the kidney to 1,25-dihydroxy vitamin D3 (calcitriol,
the most potent vitamin D metabolite) by a renal tubular mitochondrial hydroxylase: 1-α-hydroxylase.

143. A 35-year-old male undergoes resection of a nasal polyp. From an epidemiological standpoint, which of the
following clinical or pathologic features is most likely to be associated with such a polyp?
a. Asthmatic attacks on aspirin ingestion
b. Cystic fibrosis
c. Epstein-Barr virus (EBV)
d. Occupational risk

Ans: a
Ref: Robbins and Cotran Pathologic Basis of disease 9/e p735
Inflammatory nasal polyps;
• Nasal polyps can be grouped into three categories: (a) inflammatory (most common), (b) associated with
cysticfibrosis (CF), and (c) antrochoanal.
• Inflammatory polyps usually manifest in patients older than 30 years and are associated with bronchospasm on
aspirin consumption in 20% of cases. They are composed of edematous lamina propria infiltrated by
lymphocytes,eosinophils, and plasma cells.
• Nasal polyps in children should arouse suspicion of CF. Antrochoanal polyps arise from cysts of the maxillary
sinusprotruding into the nasal cavity through the maxillary meatus

144. A 30-year-old man, in ultrasound examination shows a 3.5-cm solid mass in the right testis. Laboratory
findings include markedly increased serum levels of chorionic gonadotropin and α-fetoprotein. Which of the
following testicular neoplasms is the most likely diagnosis?
a. Leydig cell tumor

Paradise Institute & Self Study Centre Page 48


PARADISE WEEKLY MODEL TEST CEE MD/MS BASED MAGH 16

b. Mixed germ cell tumor


c. Pure spermatocytic seminoma
d. Choriocarcinoma

Ans: b
Ref: Robbins and Cotran Pathologic basis of disease9/e p979
 Although a modest elevation of the human chorionic gonadotropin (hCG) concentration can occur when a
seminoma contains some syncytial giant cells, significant elevation of the α-fetoprotein (AFP) level never occurs
with pure seminomas. Elevated levels of AFP and hCG effectively exclude the diagnosis of a pure seminoma.
 The yolk sac element explains the high AFP level. Mixed tumors may include seminoma.
 Leydig cell tumors are non–germ cell tumors derived from the interstitial (Leydig) cells; they may elaborate
androgens.
 Choriocarcinomas secrete high levels of hCG, but no AFP.

145. A 27-year-old woman and her 33-year-old sister were diagnosed with infiltrating ductal carcinoma of the
breast, and both had bilateral mastectomies. Which of the following risk factors is most significant for this type of
cancer?
a. Oral contraceptive use
b. Inheritance of a mutant p53 allele
c. Obesity
d. Multiparity

Ans: b
Ref: Robbins and Cotran Pathologic basis of disease9/e p1054
Bilateral breast cancer in very young women in the same family suggests a germline mutation in a tumor suppressor gene.
The affected genes may be BRCA1, BRCA2, or p53. The BRCA1 and BRCA2 genes account for most hereditary breast
cancers. Establishment of other risk factors is not as secure. Multiparity reduces the risk of breast cancer.

146. A 66-year-old man has experienced increasing malaise for 3 years. Physical examination shows no remarkable
findings. Laboratory findings include a serum creatinine level of 4.9 mg/dL and urea nitrogen level of 45 mg/dL.
Abdominal CT scan shows bilaterally enlarged cystic kidneys. DNA analysis shows a polycystin-1 gene mutation.
Which of the following lesions is the most likely complication of this man's disease?
a. Multinodular goiter
b. Islet cell hyperplasia
c. Adrenal atrophy
d. Parathyroid hyperplasia

Ans: d
Ref: Robbins and Cotran Pathologic basis of disease9/e p1103
This patient has autosomal-dominant polycystic kidney disease with secondary hyperparathyroidism resulting from
decreased phosphate excretion by the kidneys. The resultant hyperphosphatemia depresses the serum calcium level and
stimulates parathyroid gland activity. Because of reduced renal parenchymal function, there also is less active vitamin D,
which leads to decreased dietary calcium absorption. Renal failure does not lead to any other endocrine lesions.

147. A 35-year-old woman presents with increasing fatigue, slight weight gain, and a low-grade fever.
Approximately 3 weeks before developing these symptoms she had a 4-day upper respiratory viral infection. At
present a physical examination finds a tender enlarged thyroid gland. If a biopsy specimen reveals granulomatous
inflammation with giant cells, which of the following is the correct diagnosis?
a. De Quervain thyroiditis
b. Graves disease
c. Hashimoto thyroiditis
d. Plummer disease

Ans: a

Paradise Institute & Self Study Centre Page 49


PARADISE WEEKLY MODEL TEST CEE MD/MS BASED MAGH 16

Ref: Robbins and Cotran Pathologic basis of disease9/e


De-Quervain (subacute) thyroiditis is a self-limited cause of hypothyroidism. It typically follows a viral infection when
patients develop the acute onset of fever with painful enlargement of the thyroid gland. Patients may develop mild
hyperthyroidism early, but later they usually develop hypothyroidism. De Quervain thyroiditis also is known as
granulomatous thyroiditis because histologic sections will show granulomatous inflammation with giant cells sur- rounding
fragments of colloid.

148. The clinical combination of hyperthyroidism, exophthalmos, and pretibial myxedema is most likely to be seen in an
individual with which of the following types of autoantibodies?
a. Antimitochondrial antibodies
b. Antineutrophil cytoplasmic antibodies
c. TSH-receptor-blocking antibodies
d. TSH-receptor-stimulating antibodies

Ans: d
Ref: Robbins and Cotran Pathologic basis of disease9/e p651
The clinical combination of hyperthyroidism, exophthalmos, and pretibial myxedema is most consistent with the diagnosis
of Graves disease. This autoimmune disorder is the most common cause of hyperthyroidism. It is characterized by the
presence of IgG autoanti- bodies that bind to and stimulate the TSH receptor. Patients develop diffuse uniform enlargement
of the thyroid gland; this is called a dif- fuse toxic goiter.

149. On physical examination, a female newborn is found to have a defect in the lumbosacral region through which
a segment of the spinal cord protrudes with an overlying meningeal outpouching. The mother did not receive
prenatal care or routine prenatal screening. The abnormal- ity seen in this newborn is associated with a maternal
deficiency of which of the following substances?
a. Ascorbic acid
b. Cystathionine
c. Folic acid
d. Vitamin B12

Ans: c
Ref: Robbins and Cotran Pathologic Basis of disease 9/ep
The etiology of neural tube developmental defects such as anen- cephaly and spina bifida is multifactorial in origin, but
these defects clearly are related to a maternal deficiency of folic acid. Indeed, dietary supplementation with folate during
pregnancy has been shown to decrease the incidence of neural tube defects significantly.

Pharmacology

150. The pharmacokinetics change occurring in geriatric patients is decline in:


a. Gastric absorption
b. Liver metabolism
c. Renal clearance
d. Hypersensitivity

Ans: c
Ref: KD Tripathi Essentials of Medical Pharmacology; 7th Edition, Page No: 30
Renal function again progressively declines after the age of 50 years; renal clearance of most drugs is substantially lower in
the elderly (>75 yr).

151. Which of the following classes of cancer chemotherapeutic agents bind tubulin and cause arrest of cells in
metaphase?
a. Alkylating agents
b. Antimetabolites
c. Taxanes

Paradise Institute & Self Study Centre Page 50


PARADISE WEEKLY MODEL TEST CEE MD/MS BASED MAGH 16

d. Vinca alkaloids

Ans: d
Ref: Abal M, Andreu JM, Barasoain I. Taxanes: microtubules and centrosome targets, and cell cycle dependent
mechanism of action. Curr Cancer Drug Targets 2003;3(3):193–203.
Vinca alkaloids (vinblastine, vincristine, vinorelbine) bind tubulin to terminate microtubule assembly and cause cell arrest
in metaphase (M) by blocking mitosis and chromosomal aggregation and causing mitotic spindle dissolution. Alkylating
agents form covalent bonds with adjacent guanine residues and inhibit DNA replication and tran- scription.
Antimetabolites compete with naturally occurring com- pounds for binding sites on enzymes or else become incorporated
into DNA or RNA to interfere with cell growth and division. Taxane (paclitaxel) binds to microtubules resulting in their
stabilization and in an enhancement of aberrant tubulin polymerization that result in cytotoxicity, including mitotic arrest.

152. An example of covalent drug receptor interaction is:


a. Noradrenaline binding to b1 adrenergic receptor
b. Acetylcholine binding to muscarinic receptor
c. Prazosin binding to a1 adrenergic receptor
d. Phenoxybenzamine binding to alpha adrenergic receptor

Ans: d
Ref: KD Tripathi Essentials of Medical Pharmacology; 7th Edition, Page No: 141
Phenoxybenzamine
 It cyclizes spontaneously in the body giving rise to a highly reactive ethyleniminium intermediate which reacts
with α adrenoceptors and other biomolecules by forming strong covalent bonds.
 The α blockade is of nonequilibrium (irreversible) type and develops gradually (even after i.v. injection) and lasts
for 3–4 days till fresh receptors are synthesized.

153. A 38-year-old woman develops palpitations, weight loss, and heat intolerance. On examination, she has a mild
tremor, an enlarged thyroid, and resting tachycardia. Biochemical tests confirm the diagnosis and she is started on
methimazole. Which of the following is the most likely mechanism of this drug?
a. Inhibition of iodine uptake
b. Inhibition of thyroidal organic binding and coupling reactions
c. Lowering serum calcium
d. Adrenal suppression

Ans: b
Ref:Brunton LL, Lazo JS, Parker KL. Goodman and Gilman’s The Pharmacological Basis of Therapeutics. 11th
ed. New York,NY: McGraw-Hill; 2006. Page No: 1527–1528.
• Methimazole is an effective treatment for hyperthyroidism. Methimazole interferes with thyroid function mainly
by inhibition of thyroidal organic binding and coupling reactions.
• In contrast to other agents such as perchlorate, the action of thioamides is not prevented by large doses of iodide.

154. Side effects of acetazolamide include all except:


a. Hypokalemia
b. Drowsiness
c. Diarrhea
d. Paraesthesia

Ans: c
Ref: KD Tripathi Essentials of Medical Pharmacology; 7th Edition, Page No: 586, 587
Acetazolamide
 It is a sulfonamide derivative which noncompetitively but reversibly inhibits CAse (type II) in PT cells resulting in
slowing of hydration of CO2 → decreased availability of H+ to exchange with luminal Na+ through the Na+-H+
antiporter.

Paradise Institute & Self Study Centre Page 51


PARADISE WEEKLY MODEL TEST CEE MD/MS BASED MAGH 16

 Inhibition of brush border CAse (type IV) retards dehydration of H2CO3 in the tubular fluid so that less CO2
diffuses back into the cells.
Adverse effects are frequent.
 Acidosis, hypokalaemia, drowsiness, paresthesias, fatigue, abdominal discomfort.
 Hypersensitivity reactions—fever, rashes.
 Bone marrow depression is rare but serious.
 It is contraindicated in liver disease: may precipitate hepatic coma by interfering with urinary elimination of NH3
(due to alkaline urine).
 Acidosis is more likely to occur in patients of COPD.

155. Which of the following is an indication for the use of raloxifene?


a. Chronic renal failure
b. Hypoparathyroidism
c. Renal osteodystrophy
d. Post-menopausal osteoporosis

Ans: d
Ref: KD Tripathi Essentials of Medical Pharmacology; 7th Edition, Page No: 314
Raloxifene: This SERM has a different pattern of action than tamoxifen. It is an estrogen partial agonist in bone and
cardiovascular system, but an antagonist in endometrium and breast. It has high affinity for both ERα and ERβ, and has a
distinct DNA target the ‘raloxifene response element’ (RRE).
Raloxifene is a second line drug for prevention and treatment of osteoporosis in postmenopausal women; Ca2+ and vit D
supplements enhance the benefit.

156. Which of the following has highest potential to cause metabolic syndrome?
a. Clozapine
b. Risperidone
c. Quetiapine
d. Aripiprazole

Ans: a
Ref: Metabolic Disturbances, Side Effect Profile and Effectiveness of Clozapine in Adolescents by Sandeep Grover
et al; Indian J Psychol Med. 2016 May-Jun; 38(3): 224–233.
Clozapine is effective in the management of treatment-resistant schizophrenia in adolescents and a higher level of
response at 3 months is associated with the development of metabolic syndrome later on. Among other side effects,
hypersalivation and sedation are the most common side effects of clozapine in adolescents.

157. Which of the following is the most characteristic symptom in methanol poisoning?
a. Carcinogenicity
b. Hepatic injury
c. Renal injury
d. Visual disturbances

Ans: d
Ref: KD Tripathi Essentials of Medical Pharmacology; 7th Edition, Page No: 744
The most characteristic symptom in methanol poisoning is visual disturbances. Hepatic injury, renal injury, and potential
carcinogenicity are more characteristic of the halogenated aliphatic hydrocarbons.

158. Neurotoxicity is dose-limiting for which of the following drugs?


a. Etoposide
b. Methotrexate
c. Paclitaxel
d. Vincristine

Paradise Institute & Self Study Centre Page 52


PARADISE WEEKLY MODEL TEST CEE MD/MS BASED MAGH 16

Ans: d
Ref: KD Tripathi Essentials of Medical Pharmacology; 7th Edition
Neurotoxicity is dose-limiting for vincristine. Myelosuppression is dose-limiting for paclitaxel, etoposide, and
methotrexate.
PHARMACOLOGY PEARLS
• The plant anticancer alkaloids act on the microtubules, during the mitosis (M) phase of the cell cycle.
• The vinca alkaloids cause sensory and motor toxicities with the fol- lowing order of activity: vincristine is
greater than vinblastine is greater than vinorelbine.
• Neurotoxicity is dose limiting for vincristine, whereas myelosuppres- sion is dose limiting for paclitaxel,
etoposide, and methotrexate.

159. True about orphan drug is:


a. Developed for orphans
b. Drugs used very rarely
c. Drugs used for rare diseases
d. Rare drug for common diseases

Ans: c
Ref: KD Tripathi Essentials of Medical Pharmacology; 7th Edition, Page No: 5
Orphan Drugs
 These are drugs or biological products for diagnosis/treatment/ prevention of a rare disease or condition, or a more
common disease (endemic only in resource poor countries) for which there is no reasonable expectation that the
cost of developing and marketing it will be recovered from the sales of that drug.
 The list includes sodium nitrite, fomepizole, liposomal amphotericin B, Miltefosine, rifabutin, succimer,
somatropin, digoxin immune Fab (digoxin antibody), liothyronine (T3) and many more.
 Though these drugs may be life saving for some patients, they are commercially difficult to obtain as a medicinal
product.
 Governments in developed countries offer tax benefits and other incentives to pharmaceutical companies for
developing and marketing orphan drugs.
Biochemistry

160. 2 molecules of glucose is present in:


a. Maltose
b. Lactose
c. Sucrose
d. Lactulose

Ans: a
Ref: Chattarjea Textbook of Biochemistry 8th Edition, Page No: 24
 Maltose yields 2 molecules of glucose on hydrolysis.
 Lactose yields one molecule of glucose and one molecule of galactose on hydrolysis.
 Sucrose yields one molecule of glucose and one molecule of fructose on hydrolysis.
 Lactulose a ketodisaccharide

161. A 30-month-old female child whose growth rate has been in the lower 10th percentile over the last year
presents with chronic, nonproductive cough and diarrhea with foul-smelling stools. She is diagnosed as having
cystic fibrosis. For which of the following vitamins is this child most likely to be at risk of deficiency?
a. Ascorbic acid (vitamin C)
b. Biotin
c. Retinol (vitamin A)
d. Riboflavin (vitamin B2)

Ans: c
Ref: Chattarjea Textbook of Biochemistry 8th Edition
Paradise Institute & Self Study Centre Page 53
PARADISE WEEKLY MODEL TEST CEE MD/MS BASED MAGH 16

Because cystic fibrosis leads to pancreatic damage and diminution of the ability to secrete HCO3 and pancreatic digestive
enzymes with the result that fat and protein are absorbed poorly. Retinol is a fat- soluble vitamin that must be absorbed
along with lipid micelles; other fat-soluble vitamins are E, D, and K. The other vitamins listed are water-soluble and their
absorption is not significantly affected.

162. A 27-year-old man has been rushed to the ED following his sudden collapse and entry into a state of
unconsciousness. Examination of personal belongings revealed the patient is an insulin-dependent diabetic. A rapid
decline in which of the following humoral factors likely triggered the sudden collapse of the patient?
a. Insulin
b. Glucagon
c. Fatty acids
d. Glucose

Ans: d
Ref: Chattarjea Textbook of Biochemistry 8th Edition
Glucose is the primary source of energy for the central nervous system. A sudden decrease in circulating glucose levels
will therefore impair ATP generation, in turn impeding cognitive function. If hypo- glycemia persists, the patient will slip
into a coma and eventually die. This unfortunately common complication in type I diabetics is a con- sequence of
oversupplementation with insulin. In contrast, a sudden decrease in circulating insulin, glucagon, fatty acids, or triglyceride
would have little immediate effect on cognitive function.

163. Acute Phase Proteins include:


a. Haptoglobin
b. α1-antitrypsin
c. Fibrinogen
d. All of the above

Ans: d
Ref: Chattarjea Textbook of Biochemistry 8th Edition, Page No: 103
Acute Phase Proteins (or Reactants)
Levels of certain proteins in plasma increase during acute inflammatory states or secondary to certain types of tissue
damage. These proteins are called Acute phase proteins or reactants.
They include:
 C-reactive protein (CRP)
 Haptoglobin (Hp)
 α1-antitrypsin
 α1-acid glycoprotein (orosomucoid) and
 Fibrinogen
These proteins (except fibrinogen) discussed above.

164. Metabolic fate of NH3 in the body is all except:


a. Converted to urea
b. Formation of Glutamine
c. Amination of α-ketoacid to form α-amino acid
d. Converted to xanthine

Ans: d
Ref: Chattarjea Textbook of Biochemistry 8th Edition, Page No: 479
Metabolic fate of NH3 in the body: Three main fates:
 Mainly NH3 is converted to urea (urea cycle)
 Formation of Glutamine
 Amination of α-ketoacid to form α-amino acid

165. Acid rain is due to precipitation of:

Paradise Institute & Self Study Centre Page 54


PARADISE WEEKLY MODEL TEST CEE MD/MS BASED MAGH 16

a. Nitric acid
b. Hydrochloric acid
c. Acetic acid
d. Propionic acid

Ans: a
Ref: Chattarjea Textbook of Biochemistry 8th Edition, Page No: 784
Acid rain
 Acid rain is the precipitation that contains high levels of sulphuric acid (H2SO4) and nitric acid (HNO3).
 Sulphur dioxide (SO2) produced during burning of coal and oil is oxidised to sulphur trioxide (SO3) by
atmospheric oxygen (O2). This dissolves in the moisture present in the clouds or in rain water forming sulphuric
acid.
 Similarly, nitrogen dioxide produced and released by factories and automobiles forms nitric acid with
moisture/rain water.
 Acids formed by above manner pollute rivers, streams and soil. Acid rain pollutants also decrease the fertility of
the soil.
 The acid rain pollutants often travel long distances and even pollute neighbouring towns and countries.

166. Which isoenzyme of LDH is raised in myocardial infarction?


a. LDH-1
b. LDH-2
c. LDH-3
d. LDH-4

Ans: a
Ref: Harpers Illustrated Biochemistry; 30thedition; Page 69
Lactate dehydrogenase (LDH) is a tetrameric enzyme consisting of two monomer types: H (for heart) and M (for muscle)
that combine to yield five LDH isozymes:
• HHHH (I1),
• HHHM (I2),
• HHMM (I3),
• HMMM (I4),
• MMMM (I5).
The relative proportions of each subunit in the cells of a particular organ is determined by tissue-specific patterns in the
expression of the H and M genes. Isozyme I1 predominates in heart tissue, and isozyme I5in the liver. Thus, when
LDH levels rise in blood plasma, the identity of the injured tissue can be inferred from its characteristic pattern of LDH
isozymes. In the clinical laboratory, individual isozymes can be separated by electrophoresis and detected using a coupled
assay. While historically of importance, the assay of LDH has been superseded as a marker for MI by proteins that appear
in plasma more rapidly than LDH.

167. Instant energy is given by:


a. Glucose
b. Sucrose
c. Maltose
d. Glycogen

Ans: a
Ref: Harper’s Biochemistry; 30th edition; Pg 140
In the fed state, after a meal, there is an ample supply of carbohydrate, and the metabolic fuel for most tissues is
glucose.
In the fasting state, glucose must be spared for use by the central nervous system (which is largely dependent on glucose)
and the red blood cells (which are wholly reliant on glucose). Therefore, tissues that can use fuels other than glucose do so;
muscle and liver oxidize fatty acids and the liver synthesizes ketone bodies from fatty acids to export to muscle and other
tissues. As glycogen reserves become depleted, amino acids arising from protein turnover are used for gluconeogenesis.

Paradise Institute & Self Study Centre Page 55


PARADISE WEEKLY MODEL TEST CEE MD/MS BASED MAGH 16

Microbiology

168. pH of Sabouraud’s dextrose agar is adjusted to:


a. 4-6
b. 1-2
c. 6-8
d. 8-10

Ans: c
Ref: Jawetz’s Medical Microbiology; Lange; 26th Edition, Page No: 676
Most fungi occur in nature and grow readily on simple sources of nitrogen and carbohydrate. The traditional mycological
medium, Sabouraud’s agar, which contains glucose and modified peptone (pH 7.0), has been used because it does not
readily support the growth of bacteria.
The morphologic characteristics of fungi used for identification have been described from growth on Sabouraud’s agar. To
culture medical fungi from non-sterile specimens, antibacterial antibiotics (eg, gentamicin, chloramphenicol) and
cycloheximide are added to the media to inhibit bacteria and saprobic molds, respectively.

169. Which of the following favor Gram positive bacterial cell wall?
a. Thin
b. No teichoic acid
c. No porin
d. Periplasmic space present

Ans: c
Ref: Surinder Kumar Microbiology, 1st Edition, Page No: 22
Comparison of cell walls of gram-positive and gram-negative bacteria

170. Dental caries is caused by:


a. S. gordonii
b. S. mitis
c. S. mutans
d. S. salivarius

Ans: c
Ref: Surinder Kumar Microbiology, 1st Edition, Page No: 251
Dental Caries
S. mutans is the principal cause of dental caries (tooth decay). S. mutans adheres to dental surfaces via extracellular
carbohydrates (dextran) and erodes the teeth by converting sucrose to acetic acid and lactate. It breaks down dietary
sucrose, producing acid and a tough adhesive dextran. The acid damages dentine and the dextrans bind together food
debris, epithelial cells, mucus and bacteria to form dental plaques, which lead to caries.

171. Viroids contains:


a. Circular single stranded RNAs
b. Circular single stranded RNAs with a protein coat
c. Circular single stranded RNAs with a protein coat and envelope
d. Circular single stranded RNAs with a protein coat, envelope and attachment sites

Paradise Institute & Self Study Centre Page 56


PARADISE WEEKLY MODEL TEST CEE MD/MS BASED MAGH 16

Ans: a
Ref: Surinder Kumar Microbiology, 1st Edition, Page No: 516
Viroids are small infectious agents which are circular single stranded RNAs without a protein coat. They cause disease of
plants. Viroids are agents that do not fit the definition of classic viruses.

172. PAS, Alciari blue and mucicarmine stain _____ of C. neoformans in tissue specimens.
a. Membrane
b. Nucleic acid
c. Capsule
d. Special granule in cytoplasm

Ans: c
Ref: Surinder Kumar Microbiology, 1st Edition, Page No: 676
Cryptococcosis
 Cryptococcosis is subacute or chronic infection caused by the capsulate yeast Cryptococcus neoformans.
 C. neoformans causes: Pulmonary cryptococcosis in immunocompromised hosts central nervous system (CNS)
cryptococcosis; disseminated non-pulmonary non-CNS cryptococcosis.
 Laboratory diagnosis: In unstained, wet preparations of CSF mixed with a drop of India ink or nigrosine, the
capsule can be seen as a clear halo around the yeast cells.
 A specific fungal stain such as PAS, Alciari blue and mucicarmine, stain the capsular material of C. neoformans in
tissue specimens.
 The culture of centrifuged CSF specimens confirms diagnosis of the condition.
 Cryptococcal capsular polysaccharide antigen can be detected in CSF and blood by latex agglutination and ELISA
test.

173. Which of the following is considered upper UTI?


a. Urethritis
b. Pyelonephritis
c. Cystitis
d. Prostatitis

Ans: b
Ref: Surinder Kumar Microbiology, 1st Edition, Page No: 691
Acute infections of UTI can be subdivided into two anatomic categories:
1. Lower UTI
 Urethritis
 Cystitis
 Prostatitis
Lower UTI is due to ascending infection.
2. Upper UTI
 Acute pyelitis—infection of pelvis of kidney.
 Acute pyelonephritis—infection of parenchyma of kidney.
Pyelonephritis is probably due to hematogenous infection.

174. Counterstaining in Gram stain is done with:


a. Crystal violet
b. Gentian violet
c. Acetone
d. Carbol fuchsin

Ans: d
Ref: Surinder Kumar Microbiology, 1st Edition, Page No: 758
Gram stain

Paradise Institute & Self Study Centre Page 57


PARADISE WEEKLY MODEL TEST CEE MD/MS BASED MAGH 16

Method: The staining technique consists of four steps:


 Primary staining with a basic pararosaniline (triphenyl methane) violet dye, namely crystal violet, methyl violet
or gentian violet (a mixture of the two preceding dyes);
 Application of a dilute solution of iodine;
 Decolorisation with an organic solvent such as acetone, alcohol or aniline;
 Counterstaining with a dye of contrasting color, such as carbol fuchsin, safranine or neutral red.

175. Antigen in Weil-Felix reaction is from strains of:


a. Rickettsia
b. Proteus
c. Plasmodium
d. Prion

Ans: b
Ref: Surinder Kumar Microbiology, 1st Edition, Page No: 491
Laboratory diagnosis of rickettsial diseases may be carried out by isolation of rickettsiaeand serology.
 Serology-By the heterophile Weil-Felix reaction or by specific tests using rickettsial antigens. The Weil-Felix
reaction is an agglutination test which detects anti-rickettsial antibodies in which sera are tested for agglutinins to
the antigens of certain nonmotile Proteus strains OX 19, OX 2 and OX K.
 Specific tests using rickettsial antigens-include complement fixation test, latex agglutination test and enzyme
immunoassay.
Community Medicine

176. Major foundation of epidemiological approach is:


a. Asking question
b. Making comparison
c. Both
d. None

Ans: c
Ref: Park's Textbook of Preventive and Social Medicine; 24th Edition; Page No: 58
Epidemiological approach
The epidemiological approach to problems of health and disease is based on two major foundations:
 Asking questions
 Making comparisons

177. First requisite before conducting an interview is:


a. Securing rapport
b. Probe questions
c. Establishing contact
d. Guiding the interview

Ans. c
Ref. Park 21/e p644, Park 22/e p646
• Steps of Interview:
– Establishing contact: first requisite before conducting an interview
– Starting an interview – Securing rapport
– Recall – Probe questions
– Encouragement – Guiding the interview
– Closing the interview – Report

178. A family where all of its members are playing a part inits management is known as:
a. Elementary family
b. New family
Paradise Institute & Self Study Centre Page 58
PARADISE WEEKLY MODEL TEST CEE MD/MS BASED MAGH 16

c. 3-Generation family
d. Communal family

Ans. d
Ref. Park 21/e p633, Park 22/e p635
• Communal family: Is a family where all of its members are playing a part in its management
– Is a good example of ‘division of labour’, an important function of a family.

179. What is the most cost-effective intervention for health promotion?


a. Behavior and lifestyle modification
b. Environmental modification
c. Health education
d. Nutritional intervention

Ans: c
Ref: Park's Textbook of Preventive and Social Medicine; 24th Edition; Pg: 43
 Health promotion is "the process of enabling people to increase control over, and to improve health". It is not
directed against any particular disease, but is intended to strengthen the host through a variety of approaches
(interventions). The well-known interventions in this area are:
 Health education: This is one of the most cost effective interventions. A large number of diseases could be
prevented with little or no medical intervention if people were adequately informed about them and if they were
encouraged to take necessary precautions in time.
 Environmental modifications: A comprehensive approach to health promotion requires environmental
modifications, such as provision of safe water; installation of sanitary latrines; control of insects and rodents;
improvement of housing, etc. Environmental interventions are non-clinical and do not involve the physician.
 Nutritional interventions: These comprise food distribution and nutrition improvement of vulnerable groups; child
feeding programmes; food fortification; nutrition education, etc.
 Lifestyle and behavioural changes: The conventional public health measures or interventions have not been
successful in making inroads into lifestyle reforms. The action of prevention in this case, is one of individual and
community responsibility for health, the physician and in fact each health worker acting as an educator than a
therapist.

180. Most important Essential Fatty Acid is:


a. Linoleic Acid
b. Linolenic Acid
c. Arachidonic Acid
d. Eicosapentanoic Acid

Ans. a
Ref. Park 21/e p564, Park 22/e p566
• Essential Fatty Acids (EFA): Are those that cannot be synthesized in human body; they can only be derived from
thefood
• The most important EFA is Linoleic Acid, which serves as a basis for production of other EFA
• EFA deficiency lead to 'Phrenoderma' (Toad Skin): It is characterized by rough rash like eruptions on the back
andsides of arms and legs, the back, and the buttocks. It can be cured by giving 'linseed of safflower oil' which are rich
in
• Dietary sources of EFA:

Paradise Institute & Self Study Centre Page 59


PARADISE WEEKLY MODEL TEST CEE MD/MS BASED MAGH 16

181. All are included in Kangaroo Mother Care except:


a. Skin to skin contact
b. Early discharge and follow up
c. Free nutritional supplements
d. Exclusive Breast feeding

Ans. c
Ref. Park 22/e p496
KANGAROO MOTHER CARE for prevention of neonatal hypothermia in low birth weight/premature newborns:
- Skin-to-skin positioning of newborn on mother’s chest
- Adequate nutrition through breast feeding
- Early discharge and ambulatory care
- Support for mother and family for childcare

Research Methodology

182. Statistical Power of a trial is equal to:


a. 1 + 
b. 1 – 
c.  + 
d.  / 

Ans. b
Ref. Simple Biostatistics by Indrayan&Indrayan, 1/e p157
• Statistical Power of a test:
• Is probability of rejecting a Null hypothesis when a predetermined clinically significant difference is indeedpresent
• Power can be increased by increasing the no. of subjects in a trial
• Is the ‘probability that a study/trial will be able to detect a specified difference’
• Measures the ability to demonstrate an association when one really exists
• Power of a statistical test = 1 – Probability of Type II error = 1 – 
• Power can be increased by: Including a higher no. of subjects under trial
• There is no mathematical relationship between a and b.

183. What is commonly occurring value in a distribution of data known as?


a. Mean
b. Median
c. Mode
d. Standard deviation

Ans: c
Ref: Park's Textbook of Preventive and Social Medicine; 23rd Edition; Page No: 847 - 848
 The mode is the commonly occurring value in a distribution of data. It is the most frequent item or the most
"fashionable" value in a series of observations.
 The arithmetic mean is widely used in statistical calculation. It is sometimes simply called Mean. To obtain the mean,
the individual observations are first added together, and then divided by the number of observations. The operation of
adding together is called 'summation'.
 The median is an average of a different kind, which does not depend upon the total and number of items. To obtain the
median, the data is first arranged in an ascending or descending order of magnitude, and then the value of the middle
observation is located, which is called the median.
 The standard deviation is the most frequently used measure of deviation. In simple terms, it is defined as "Root Means
Square Deviation."

Paradise Institute & Self Study Centre Page 60


PARADISE WEEKLY MODEL TEST CEE MD/MS BASED MAGH 16

184. What is number of live births per 1000 women in the reproductive age-group (15-44 or 49 years) in a given
year also known as?
a. Age specific fertility rate
b. General fertility rate
c. General marital fertility rate
d. Gross reproduction rate

Ans: b
Ref: Park PSM, 23rdedition, page 488 – 489
 Birth rate is the simplest indicator of fertility and is defined as "the number of live births per 1000 estimated mid-year
population, in a given year".
 General fertility rate is the "number of live births per 1000 women in the reproductive age-group (15-44 or 49 years) in
a given year".
 General marital fertility rate is the "number of live births per 1000 married women in the reproductive age group (15-
44 or 49) in a given year".
 Age-specific fertility rate, defined as the "number of live births in a year to 1000 women in any specified age-group".
 Age-specific marital fertility rate is the number of live births in a year to 1000 married women in any specified age
group.
 Total fertility rate represents the average number of children a woman would have if she were to pass through her
reproductive years bearing children at the same rates as the women now in each age group. It is computed by summing
the age-specific fertility rates for all ages; if 5-year age groups are used, the sum of the rates is multiplied by 5.
 Total marital fertility rate is the average number of children that would be born to a married woman if she experiences
the current fertility pattern throughout her reproductive span.
 Gross reproduction rate is the average number of girls that would be born to a woman if she experiences the current
fertility pattern throughout her reproductive span (15-44 or 49 years), assuming no mortality.
 Net Reproduction Rate (NRR) is defined as the number of daughters a newborn girl will bear during her lifetime
assuming fixed age-specific fertility and mortality rates.

185. In a particular trial, the association of lung cancer with smoking is found to be 40% in one sample and 60% in
another. What is the best test to compare the results?
a. Chi Square Test
b. Fischer Test
c. Paired t Test
d. ANOVA Test

Ans. a
Ref. Simple Biostatistics by Indrayan&Indrayan, 1/e p170-72 and Methods in Biostatistics byMahajan, 7/e p154-
169; Park 21/e p791, Park 22/e p795]
Chi-Square Test (c2 – Test):
• Is a ‘non-parametric test’ of significance
• Is used to ‘test significance of association between 2 or more qualitative characteristics’
• Is used to compare proportions in 2 or more groups
• Is used for nonNormal (non –Gaussian) distributions
• Applications of Chi-square test:
o Test of proportions
o Test of association
o Test of goodness of fit
• Essential requirements for calculation of Chi-square test:
o Random sample
o Qualitative data
o Lowest expected frequency not < 5
• In the given question, in a particular trial, the association of lung cancer with smoking is found to be 40% in one
sampleand 60% in another, Sincw two proportions are to be compared best test will be Chi-square test.

Paradise Institute & Self Study Centre Page 61


PARADISE WEEKLY MODEL TEST CEE MD/MS BASED MAGH 16

Forensic Medicine

186. Patient’s relative gives a history of tattoo, however it was not found during autopsy. What should be dissected to find it:
a. Lymph node
b. Skin
c. Spleen
d. Kidney

Ans: a
Ref: Gautam Biswas Forensic Medicine, 3rdEdition, Page No: 79
Since some pigment migrates from the tattoo site to the body’s lymph nodes, pigmentations of the axillary lymph nodes in
upper extremities tattoos could be identified with the naked eye during autopsy.

187. Stack method of dental age estimation is used for:


a. Infants
b. Adults
c. Elderly
d. Adolescents

Ans: a
Ref: Gautam Biswas Forensic Medicine, 3rdEdition, Page No: 70
Stack’s method: Stack evolved a method to estimate the age from the weight of the erupting teeth of fetus and infant. He
provided a regression line of weight of growing dental tissues against age (from 5 months in utero to postnatal age of 7
months).

188. Tissue bridges are seen in:


a. Abrasion
b. Contusion
c. Laceration
d. Stab wound

Ans: c
Ref: Gautam Biswas Forensic Medicine, 3rdEdition, Page No: 195
Laceration is the tearing or splitting of skin, mucous membranes, muscles or internal organs caused by either a shearing or
a crushing force, and produced by application of a blunt force to a broad area of the body.
 Margins: Ragged, irregular and uneven; may show tearing of the extremities at angles diverging from the main
laceration, the so-called ‘shallow tails’; pieces of tissue are attached in between called tissue tags.
 Site: Occurs most commonly over bony prominences, such as the head where the skin is fixed, and easily stretched
and torn.
 Bruising and abrasion: Seen around the margin.
 Edges: May give an indication of direction in which the blow or force was applied.
 Depth of wound: Shows bridges of irregularly torn fibrous tissue, blood vessels and nerves across the interior of
the wound.
 Soiling of wound: Mud, wood splinters, sand, glass fragments or paint material of the vehicle involved, hair or
fibers may get embedded in the wound, and are of great medico-legal importance.
 Hair bulbs: Crushed.
 Hemorrhage: Less, because the arteries are crushed and torn across irregularly; they retract and blood clots
readily, except in the scalp where the temporal arteries bleed freely as they are firmly bound and unable to contract.
 Shape: May correspond with the weapon or object which produced them.
 Gaping: Seen due to pull of elastic and muscular tissues.

189. Kernohan-Woltman sign (notch) is seen in:

Paradise Institute & Self Study Centre Page 62


PARADISE WEEKLY MODEL TEST CEE MD/MS BASED MAGH 16

a. Uncal herniation
b. Central herniation
c. Transfalcial herniation
d. Foraminal herniation

Ans: a
Ref: Gautam Biswas Forensic Medicine, 3rdEdition, Page No: 251
 An intracranial mass lesion is usually associated with a contralateral hemiplegia due to either cortical dysfunction
or compression of the ipsilateral cerebral peduncle. However, a supratentorial mass lesion can cause shift of the
midbrain to the opposite side.
 The contralateral cerebral peduncle can then impinge on the tentorium cerbelli, causing the unexpected finding of
an ipsilateral hemiplegia.
 At autopsy, this can be seen as the Kernohan-Woltman notch indentating the midbrain. This can be seen in SDH,
EDH and cerebral tumors with midline shift. The process explains how clinical signs may appear on the same side
of damaged cerebral hemisphere and has been referred to as false localizing sign.

190. Most common site of injury in under water blast:


a. Tympanic membrane
b. GIT
c. Liver
d. Lung

Ans: a
Ref: Gautam Biswas Forensic Medicine, 3rdEdition, Page No: 290
Injury to gas-filled viscera is more common in underwater explosions than in air blasts. Although the colon is most
commonly affected, perforation of the stomach, small intestine and cecum are also seen.

Mandatory CPD Topics

191. Medical practitioners should keep and maintain confidential medical records in a safe location for at least:
a. 6 months
b. 1 year
c. 5 years
d. 20 years

Ans: c
A medical practitioner should maintain or demand to maintain confidential medical records of her/his patients’ history,
physical findings, investigations, treatment, and clinical progress in both private and government settings. It may be
handwritten, printed, or electronically generated. Special medical records in the form of audio and visual recording can also
be maintained.
 Medical practitioners should keep and maintain such confidential medical records in a safe location for at least 5
years from the date of last visit of the patient.
 Medical practitioners should be able to provide such medical records upon the request of patient, in response to a
judicial order, or from the order of Nepal Medical Council or any other authorized body

192. Which of the following is considered professional misconduct?


a. Not signing the prescription
b. Not mentioning the name in prescription
c. Not mentioning the NMC number in prescription
d. All of the above

Ans: d
Rational prescription and labelling of dispensed medicines

Paradise Institute & Self Study Centre Page 63


PARADISE WEEKLY MODEL TEST CEE MD/MS BASED MAGH 16

 Every medical practitioner should prescribe medicine to a patient only if and when the drug treatment is necessary
for patient’s benefit. Proper explanation should be made in plain language, and patient and/or patient family should
be explained about the dose, route of administration and duration of treatment. Likewise, potential adverse effects
of the medicines and the need for follow up should also be discussed. It is highly important that such prescriptions
be written in capital letters or printed or written in entirely legible handwriting.
 The medical practitioner should sign the prescription and should also mention her/his name and Nepal Medical
Council Registration Number. Failure to sign and mention her/his name and Nepal Medical Council registration
number is a professional misconduct.
 A medical practitioner should not run a pharmacy for dispensing drugs prescribed by other medical practitioners.
 A medical practitioner who also dispenses medicine (prescribed by himself ) to her/his patients has the personal
responsibility to ensure that the drugs are dispensed strictly in accordance with the prescription, are not
outdated/expired and are properly labelled before they are handed over to the patients. The medical practitioner
should establish suitable procedures for ensuring that drugs are properly labelled and dispensed.
 Medicines should be prescribed in generic name as far as possible.

193. Which of the following is considered against code of medical ethics?


a. Publishing professional service information in practice website
b. Publishing your educational qualification in practice website
c. Post medical activities in Facebook with an intent of procuring patients
d. All of the above

Ans: c
 A medical practitioner may publish his professional service information in either his practice website or the
website of medical practice group.
 The website may carry only the service information which is permitted by the council.
 Medical practitioner should not post her/his medical activities in social medias (like Facebook) with an intent of
procuring patients or with a sense of self-grandiosity as it is against the code of medical ethics.

194. How deep should chest compressions be for an adult victim?


a. At least 1 inch (2.5 cm) deep
b. At least 2 inches (5 cm) deep
c. At least 5 inches (7.5 cm) deep
d. At least 7 inches (10 cm) deep

Ans: b
High-quality CPR
The following are considered essential elements of high-quality CPR:
 Compression rate of 100-120/min
 Compression depth to at least one third of the anterior-posterior diameter of the chest (approximately 4 cm in
infants to 5 inches in children); for adolescents, the adult compression depth of at least 5 cm, but no more than 6
cm should be used.
 Complete chest recoil after each compression
 Minimized interruptions in chest compressions
 Avoidance of excessive ventilation

195. Which of the following is recommended for lay rescuers in CPR?


a. Complete CPR
b. COCPR
c. BOCPR
d. Any of the above

Ans: b
For lay rescuers, compression-only CPR (COCPR) is recommended.
Positioning for CPR is as follows:

Paradise Institute & Self Study Centre Page 64


PARADISE WEEKLY MODEL TEST CEE MD/MS BASED MAGH 16

 CPR is most easily and effectively performed by laying the patient supine on a relatively hard surface, which
allows effective compression of the sternum
 Delivery of CPR on a mattress or other soft material is generally less effective
 The person giving compressions should be positioned high enough above the patient to achieve sufficient leverage,
so that he or she can use body weight to adequately compress the chest

196. Which surface disinfectants are effective against COVID-19 in non-health care setting environments?
a. Sodium hypochlorite (bleach / chlorine) may be used at a recommended concentration of 0.1% or 1,000ppm (1 part
of 5% strength household bleach to 49 parts of water)
b. Alcohol at 70-90% can also be used for surface disinfection
c. Surfaces must be cleaned with water and soap or a detergent first to remove dirt, followed by disinfection
d. All of the above

Ans: d
Which surface disinfectants are effective against COVID-19 in non-health care setting environments?
 In non-health care settings, sodium hypochlorite (bleach / chlorine) may be used at a recommended concentration
of 0.1% or 1,000ppm (1 part of 5% strength household bleach to 49 parts of water). Alcohol at 70-90% can also be
used for surface disinfection. Surfaces must be cleaned with water and soap or a detergent first to remove dirt,
followed by disinfection. Cleaning should always start from the least soiled (cleanest) area to the most soiled
(dirtiest) area in order to not spread the dirty to areas that are less soiled.
 All disinfectant solutions should be stored in opaque containers, in a well-ventilated, covered area that is not
exposed to direct sunlight and ideally should be freshly prepared every day.
 In indoor spaces, routine application of disinfectants to surfaces via spraying is not recommended for COVID-19.
If disinfectants are to be applied, these should be via a cloth or wipe which is soaked in the disinfectant.

197. Irrational use of medicines include:


a. Use of more than 3 medicines per patient
b. Experience and evidence-based prescription
c. Adequate dosage of drug
d. Use of injections when oral formulations would be more appropriate

Ans: d
Examples of irrational use of medicines include: use of too many medicines per patient ("poly-pharmacy"); inappropriate
use of antimicrobials, often in inadequate dosage, for non-bacterial infections; over-use of injections when oral
formulations would be more appropriate; failure to prescribe in accordance with clinical guidelines; inappropriate self-
medication, often of prescription-only medicines; non-adherence to dosing regimes.

198. Sputum can be disinfected by all except:


a. Cresol
b. Boiling
c. Chlorhexidine
d. Autoclave

Ans: c
Sputum
 This is best received in gauze or paper handkerchiefs and destroyed by burning.
 If the amount is considerable (asin TB hospitals), it may be disinfected by boiling or autoclaving for 20 minutes
at 20 lbs pressure.
 Alternatively, the patient may be asked to spit in a sputum cup half filled with 5 per cent cresol. When the cup is
full, it is allowed to stand for an hour and the contents may be emptied and disposed off.

199. You are nearing the end of a very tedious shift when you realize that medication you administered to your last
patient was incorrect resulting in a mild overdose. What do you do?

Paradise Institute & Self Study Centre Page 65


PARADISE WEEKLY MODEL TEST CEE MD/MS BASED MAGH 16

a. Consult a senior colleague regarding how to proceed as this is the first time that you have made an error of this
type
b. Inform the ward nurse that everything is fine
c. Inform your Consultant the following day
d. Bring up the error in your next ward meeting

Ans: a
Question represents a scenario where a patient’s safety may be at risk from a significant event. We must act immediately to
ensure that the patient is well and stable. Once we have removed or dealt with the risk to the patient we can then take steps
to prevent the incident from happening again.

200. You are working in a specialized hospital to treat COVID 19 patients. At your morning briefing you are
informed by Infection Control that all hospital staff must roll their sleeves up when they have any clinical
interaction with patients. During your shift you notice that your colleague always has her sleeves down. What will
you do?
a. Tell Infection Control that your colleague is not complying with their policy
b. Speak directly to your colleague about your observation
c. Raise your observation with to the consultant in charge of the ward
d. Do not say anything immediately but monitor the situation over the course of the next few days

Ans: b
This question is looking at your communication with team members and patient focus. All doctors have a duty to raise
concerns where they believe that patient safety is being compromised by the practice of colleagues. However, doctors
strive to provide the best care possible to their patients and this situation may have arisen out of some misunderstanding or
your colleague may be concerned about exposing their forearms. It is best therefore to speak directly to your colleague to
explore the issue.

Paradise Institute & Self Study Centre Page 66

You might also like